Продажа квадроциклов, снегоходов и мототехники
second logo
Пн-Чт: 10:00-20:00
Пт-Сб: 10:00-19:00 Вс: выходной

+7 (812) 924 3 942

+7 (911) 924 3 942

Содержание

Принцип работы сцепления

Принцип работы сцепления

Сцепление является важным конструктивным элементом трансмиссии автомобиля. Сцепление предназначено для кратковременного отсоединения двигателя от трансмиссии и плавного их соединения при переключении передач, а также предохранения элементов трансмиссии от перегрузок и гашения колебаний. Сцепление автомобиля располагается между двигателем и коробкой передач.

  • Принцип работы сцепления
  • Схема однодискового сцепления
  • Схема двухдискового сцепления
  • Принцип функционирования
  • Что входит в комплект
  • Передача крутящего момента
  • Принцип работы сцепления. Устройство сцепления автомобиля
  • Характеристика элемента
  • Назначение
  • Классификация по связи ведущих и ведомых частей
  • По типу создания нажимных усилий
  • По типу привода
  • Принцип работы сцепления с механическим приводом
  • Принцип работы сцепления с гидравлическим приводом

В зависимости от конструкции различают следующие типы сцепления:

✔фрикционное сцепление

✔гидравлическое сцепление;

✔электромагнитное сцепление.

Фрикционное сцепление передает крутящий момент за счет сил трения. В гидравлическом сцеплении связь обеспечивается за счет потока жидкости. Электромагнитное сцепление управляется магнитным полем.

Самым распространенным типом сцепления является фрикционное сцепление.

Различает следующие виды фрикционного сцепления:

✔однодисковое сцепление;

✔двухдисковое сцепление;

✔многодисковое сцепление.

В зависимости от состояния поверхности трения сцепление может быть сухое и мокрое. В сухом сцеплении используется сухое трение между дисками. Мокрое сцепление предполагает работы дисков в жидкости.

На современных автомобилях устанавливается в основном сухое однодисковое сцепление.

Однодисковое сцепление имеет следующее устройство:

✔маховик;

✔картер сцепления;

✔нажимной диск;

✔ведомый диск;

✔диафрагменная пружина;

✔подшипник выключения сцепления;

✔муфта выключения;

✔вилка сцепления.

Схема однодискового сцепления

Маховик устанавливается на коленчатом вале двигателя. Он выполняет роль ведущего диска сцепления . На современных автомобилях применяется, как правило, двухмассовый маховик. Такой маховик состоит из двух частей, соединенных пружинами. Одна часть соединена с коленчатым валом, другая — с ведомым диском. Конструкция двухмассового маховика обеспечивает сглаживание рывков и вибраций коленчатого вала. В картере сцепления размещаются конструктивные элементы сцепления. Картер сцепления крепиться болтами к двигателю.

Нажимной диск прижимает ведомый диск к маховику и при необходимости освобождает его от давления. Нажимной диск соединен с корпусом (кожухом) с помощью тангенциальных пластинчатых пружин. Тангенциальные пружины, при выключении сцепления, выполняют роль возвратных пружин.

На нажимной диск воздействует диафрагменная пружина, обеспечивающая необходимое усилие сжатия для передачи крутящего момента. Диафрагменная пружина наружным диаметром опирается на края нажимного диска. Внутренний диаметр пружины представлен упругими металлическими лепестками, на концы которых воздействует подшипник выключения сцепления. Диафрагменная пружина закреплена в корпусе. Для закрепления используются распорные болты или опорные кольца.

Нажимной диск, диафрагменная пружина и корпус образуют единый конструктивный блок, который носит устоявшееся название корзина сцепления. Корзина сцепления имеет жесткое болтовое соединение с маховиком. По характеру работы различают два типа корзин сцепления — нажимного и вытяжного действия. В распространенной корзине сцепления нажимного действия лепестки диафрагменной пружины при выключении сцепления перемещаются к маховику. В вытяжной корзине сцепления наоборот — лепестки диафрагменной пружины перемещаются от маховика. Данный тип корзины сцепления характеризуется минимальной толщиной, поэтому применяется в стесненных условиях.

Ведомый диск располагается между маховиком и нажимным диском. Ступица ведомого диска соединяется шлицами с первичным валом коробки передач и может перемещаться по ним. Для обеспечения плавности включения сцепления в ступице ведомого диска размещены демпферные пружины, выполняющие роль гасителя крутильных колебаний.

На ведомом диске с двух сторон установлены фрикционные накладки. Накладки изготавливаются из стеклянных волокон, медной и латунной проволоки, которые запрессованы в смесь из смолы и каучука. Такой состав может кратковременно выдерживать температуру до 400°С. Накладки ведомого диска могут иметь и более высокую тепловую характеристику. На спортивных автомобилях устанавливают т.н. керамическое сцепление, накладки ведомого диска которого состоят из керамики, кевлара и углеродного волокна. Еще более прочные металлокерамические накладки, выдерживающие температуру до 600°С.

Подшипник выключения сцепления (обиходное название — выжимной подшипник) является передаточным устройством между сцеплением и приводом. Он располагается на оси вращения сцепления и непосредственно воздействует на лепестки диафрагменной пружины. Подшипник располагается на муфте выключения. Перемещение муфты с подшипником обеспечивает вилка сцепления.

Схема двухдискового сцепления

На грузовых и легковых автомобилях с мощным двигателем применяется двухдисковое сцепление. Двухдисковое сцепление осуществляет передачу большего крутящего момента при неизменном размере, а также обеспечивает больший ресурс конструкции. Это достигнуто за счет применения двух ведомых дисков, между которыми установлена проставка. В результате получены четыре поверхности трения.Принцип работы сцепления

Однодисковое сухое сцепление постоянно включено. Работу сцепления обеспечивает привод сцепления.

При нажатии на педаль сцепления привод сцепления перемещает вилку сцепления, которая воздействует на подшипник сцепления. Подшипник нажимает на лепестки диафрагменной пружины нажимного диска. Лепестки диафрагменной пружины прогибаются в сторону маховика, а наружный край пружина отходит от нажимного диска, освобождая его. При этом тангенциальные пружины отжимают нажимной диск. Передача крутящего момента от двигателя к коробке передач прекращается.

При отпускании педали сцепления диафрагменная пружина приводит нажимной диск в контакт с ведомым диском и через него в контакт с маховиком. Крутящий момент за счет сил трения передается от двигателя к коробке передач.

Принцип функционирования

Прежде всего, взаимодействие между двигателем, сцеплением и коробкой передач необходимо для того, чтобы автомобиль мог беспрепятственно двигаться и останавливаться в требуемой точке. Впервые прообраз сцепления стал применяться создателями Мерседеса. Это позволило значительно упростить управление транспортным средством, поэтому сегодня работа автомобиля немыслима без этого важнейшего узла.

Итак, главный принцип работы устройства заключается в соединении первичного трансмиссионного вала и маховика силового агрегата. Благодаря такой схеме удается достичь плавности хода и переключения скоростей в коробке. Без сцепления затруднительно было бы трогаться с места. Оно устанавливается между коробкой передач и силовым агрегатом и дает возможность передавать крутящий момент от движка на колеса и, при необходимости, разрывать эту связь.

Однодисковое сцепление, как и другие его разновидности, подвержено серьезным нагрузкам в процессе эксплуатации. Многие из его составляющих требуют профилактики и своевременной замены. Неумелые и неопытные водители зачастую «палят» сцепление, и это выражение имеет под собой не только переносный смысл, поскольку в салоне автомобиля начинает ощущаться характерный запах гари.

Что входит в комплект

  • диск сцепления, обладающий характерной круглой формой, включающий несколько основных элементов;
  • диск нажимной (корзина) — его основание включает в себя пружины, совмещенные с прижимной платформой и компактно размещенные. В основании этого узла действует выжимной подшипник;

  • подшипник выжимной, отвечает за механический привод в действие вилки, и размещается на первичном валу коробки передач;
  • маховик.

Передача крутящего момента

Ведомый диск постоянно зафиксирован вместе с маховиком при помощи диска нажимного. Чтобы автомобиль тронулся, ведомый диск должен соприкоснуться с маховиком, который вращается.

Происходит это так: водитель выжимает педаль сцепления, что позволяет ему включить 1‑ю скорость. Как только педаль отпускается, пружины диска нажимного соединяют ведомый диск с маховиком. Вследствие этого касания машина начинает постепенно двигаться. Скорость вращения диска и маховика постепенно выравнивается, чем и достигается движение транспортного средства.

Полностью крутящий момент передается тогда, когда выравниваются скорости вращения ведомого диска, диска сцепления и маховика. Если отпустить педаль слишком резко, машина может попросту заглохнуть — этим часто грешат начинающие водители. При переключении любой передачи, необходимо добиваться плавного хода педали, что позволит продлить срок эксплуатации этого узла, да и трансмиссии тоже.

Принцип работы сцепления. Устройство сцепления автомобиля

Сцепление – неотъемлемая часть любого современного автомобиля. Именно этот узел принимает на себя все колоссальные нагрузки и удары. Особенно высокое напряжение испытывают устройства на автомобилях с механической КПП. Как вы уже поняли, в сегодняшней статье мы рассмотрим принцип работы сцепления, его конструкцию и назначение.

Характеристика элемента

Сцепление представляет собой силовую муфту, которая осуществляет передачу крутящего момента между двумя основными составляющими автомобиля: двигателем и коробкой передач. Состоит оно из нескольких дисков. В зависимости от типа передачи усилий данные муфты могут быть гидравлическими, фрикционными или же электромагнитными.

Назначение

Автоматическое сцепление предназначено для временного отсоединения трансмиссии от двигателя и плавной их притирки. Необходимость в ней возникает по мере того, как начинается движение. Временное разъединение мотора и КПП нужно и при последующем переключении скоростей, а также при резком торможении и остановке транспортного средства.

Во время движения машины система сцепления находится по большей части во включенном состоянии. В это время она передает мощность от двигателя к коробке переключения передач, а также предохраняет механизмы КПП от различных динамических нагрузок. Тех, которые возникают в трансмиссии. Таким образом, нагрузки на нее возрастают по мере торможения двигателя, при резком включении сцепления, снижении частоты оборотов коленвала либо при наезде транспортного средства на неровности дорожного полотна (ямы, выбоины и так далее).

Классификация по связи ведущих и ведомых частей

Сцепление классифицируют по нескольким признакам. По связи ведущих и ведомых частей принято различать следующие типы устройств:

  • Фрикционные.
  • Гидравлические.
  • Электромагнитные.

По типу создания нажимных усилий

По данному признаку различают типы сцепления:

  • С центральной пружиной.
  • Центробежные.
  • С периферийными пружинами.
  • Полуцентробежные.

По количеству ведомых валов системы бывают одно-, двух- и многодисковые.

По типу привода

  • Механический.
  • Гидравлический.

Все вышеуказанные типы сцеплений (за исключением центробежных) являются замкнутыми, то есть постоянно выключенными или включенными водителем при переключении скоростей, остановке и торможении транспортного средства.

На данный момент большую популярность обрели системы фрикционного типа. Такие узлы используются как на легковых, так и на грузовых автомобиля, а также на автобусах малого, среднего и большого класса.

2-дисковые сцепления используются только на крупнотоннажных тягачах. Также они устанавливаются на автобусы большой вместимости. Многодисковые же практически не применяются автопроизводителями в данный момент. Раньше они использовались на большегрузах. Также стоит отметить, что гидромуфты в качестве отдельного узла на современных машинах не применятся. До недавнего времени они использовались в коробках автомобилей, однако только совместно с последовательно установленным фрикционным элементом.

Что касается электромагнитных сцеплений, то они на сегодняшний день не получили широкого распространения в мире. Связано это со сложностью их конструкции и с дорогостоящим обслуживанием.

Принцип работы сцепления с механическим приводом

Стоит отметить, что данный узел имеет одинаковый принцип работы вне зависимости от количества ведомых валов и типа создания нажимных усилий. Исключение составляет тип привода. Напомним, он бывает механическим и гидравлическим. И сейчас мы рассмотрим принцип работы сцепления с механическим приводом.

Как же действует данный узел?

В рабочем состоянии, когда педаль сцепления не затронута, ведомый диск зажат между нажимным и маховиком.

В это время передача крутящих усилий на вал производится за счет силы трения.

Когда водитель нажимает ногой на педаль, трос сцепления перемещается в корзине. Далее рычаг поворачивается относительно своего места крепления. После этого свободный конец вилки начинает давить на выжимной подшипник.

Последний, перемещаясь к маховику, — давить на пластины, которые отодвигают нажимной диск. В данный момент ведомый элемент освобождается от прижимающих усилий и таким образом происходит отсоединение сцепления.

Далее водитель свободно производит переключение передачи и начинает плавно отпускать педаль сцепления. После этого система вновь включает в связь ведомый диск с маховиком. По мере отпускания педали сцепление включается, происходит притирка валов. Через некоторое время (пару секунд) узел в полной мере начинает передавать крутящий момент на двигатель.

Последний через маховик осуществляет привод на колеса. Стоит отметить, что трос сцепления присутствует только на узлах с механическим приводом. Нюансы конструкции другой системы мы опишем в следующем разделе.

Принцип работы сцепления с гидравлическим приводом

Здесь, в отличие от первого случая, усилие от педали к механизму передается посредством жидкости.

Последняя содержится в специальных трубопроводах и цилиндрах.

Устройство данного типа сцепления несколько отличается от механического.

На шлицевом конце ведущего вала трансмиссии и стального кожуха, закрепленного к маховику, устанавливается 1 ведомый диск.

Внутри кожуха есть пружина с радиальным лепестком. Она служит выжимным рычагом. Управляющая педаль при этом подвешивается на оси к кронштейну кузова.

К ней также прикреплен толкатель главного цилиндра на шарнирном соединении. После того как происходит выключение узла и переключение передачи, пружина с радиальными лепестками возвращает педаль в исходное положение.

В конструкции узла присутствует как главный, так и рабочий цилиндр сцепления. По своей конструкции оба элемента очень схожи между собой. Оба состоят из корпуса, внутри которого присутствует поршень и специальный толкатель. Как только водитель нажимает педаль, задействуется главный цилиндр сцепления. Здесь при помощи толкателя поршень перемещается вперед, благодаря чему давление внутри увеличивается. Последующее его передвижение приводит к тому, что жидкость проникает в рабочий цилиндр через нагнетательный канал. Так вот, благодаря воздействию толкателя на вилку и происходит выключение узла. В то время, когда водитель начинает отпускать педаль, рабочая жидкость поступает обратно. Это действие приводит к включению сцепления. Данный процесс можно описать так. Сначала открывается обратный клапан, который сжимает пружину.

Далее идет возврат жидкости из рабочего цилиндра в главный. Как только давление в нем становится меньше усилия нажатия пружины, клапан закрывается, а в системе образуется избыточное давление жидкости. Так происходит нивелирование всех зазоров, которые находятся в определенной части системы.

Понравилась статья? Расскажите друзьям:

Оцените статью, для нас это очень важно:

Проголосовавших: 1 чел.
Средний рейтинг: 5 из 5.

назначение, устройство и принцип действия

В конструктивном плане сцепление может иметь значительные отличия, но служит для достижения одинаковых целей. Этот узел на авто предназначен для отключения крутящего момента, передаваемого от силового агрегата непосредственно на трансмиссию. Потребность в размыкании сцепления возникает у водителя при переходе на другую передачу. Узел относится к разряду высоконагруженных. Срок службы зависит от условий эксплуатации машины, от умения, навыков человека за рулем и конечно же, от качества комплектующих.

Устройство сцепления

Вкратце обозначим принцип действия. Рабочий процесс происходит при давлении на педаль. В гидравлической системе автомобиля нарастает давление, начинает работу вилка, передающая усилие на выжимной подшипник. С его помощью, в свою очередь, происходит давление на диафрагменную пружину. В результате происходит размыкание в механизме, во время которого меняется передача в КПП. После того, как педаль спокойно отпускается, диски в плавном режиме смыкаются, благодаря чему происходит передача момента непосредственно на вал коробки. Плавность переключения достигается за счет умелых действий водителя и благодаря наличию фрикционной накладки, которая в момент включения подвергается износу.

Привод выключения сцепления

Механические конструкции давно не используются и рассматривать их нет смысла. Классическая современная система сцепления предусматривает передачу усилия за счет жидкости, не подлежащей сжатию. Представим комплектацию узла автомобиля, работающего на основе гидравлики. Сюда входят педаль, а также главный и рабочий цилиндры. Потребуется магистраль и емкость с рабочей жидкостью. Механизм срабатывает после создания давления в системе, которое образуется при нажатии на педаль. За счет давления на поршень в действие вступает шток, толкающий, в свою очередь, вилку. В исходное положение возврат происходит при отпускании педали. На магистральных тягачах, других видах грузовых авто и тяжелой спецтехнике дополнительно применяются усилители, облегчающие работу водителя, оператора.

Механизм сцепления

Именно здесь передается крутящий момент, параметры которого могут достигать колоссальных значений на карьерных самосвалах. Комплектация включает:

  • Картер, кожух;
  • В качестве ведущего диска выступает маховик ДВС;
  • Нажимной диск;
  • Ведомый диск, на котором есть устойчивые к значительным нагрузкам накладки

Виды сцепления

Разнообразие конструкторских разработок позволяет выделить более десятка видов механизмов, существенно отличающихся по принципу действия. Существуют различия по типу управления и видам трения, по характеристикам включения и по количеству ведомых дисков. Мы же сосредоточим внимание на самых популярных, востребованных инженерных решениях.

Сухое сцепление

Этот тип механизмов на автомобилях остается самым востребованным, обладающим большим ресурсом. Механизмы активно используют для комплектации самых разных типов машин, включая легковые, грузовые, а также спецтехнику. Основным отличием сухого сцепления считается полное отсутствие жидкостей на рабочих поверхностях. В результате обеспечивается жесткий контакт между мотором и КПП. Более того, попадание разного рода жидкостей на рабочие поверхности крайне нежелательно, так как, сказывается на характеристиках узла.

Мокрое сцепление

Преимущества механизма заключаются в наличии среды, благодаря которой происходит охлаждение комплектующих. Сцепление подобного типа подразумевает комплектацию механизма масляной емкостью, гидронасосом. Крутящий момент быстро подается на трансмиссию посредством сжатия обоих дисков. Механизмы автомобилей рассчитаны на передачу солидного крутящего момента, отличаются высокой надежностью, но не лишены и недостатков. Это сложность инженерных решений, высокая стоимость узла. Самыми распространенными механизмами подобного типа можно считать DSG

Сухое двухдисковое сцепление

Здесь принципиально другой принцип работы комплектующих, не предполагающий наличие масляной ванны. К преимуществам можно отнести невысокую стоимость, а также вы уровень КПД. Дело в том, что отсутствие масляного насоса позволяет трансмиссии функционировать с минимальным уровнем потерь. Если говорить о недостатках, то это существенно меньшие возможности по передаче крутящего момента н автомобиле. Справедливости ради отметим, что первые «сухие» DSG были достаточно «сырыми». Поломки нередко происходили при пробеге 20-40 тыс. км.

Сцепление двухмассового маховика

Производители современных автомобилей широко используют сложный механизм для того, чтобы сгладить вибрационные нагрузки, поступающие от силового агрегата и значительные крутильные колебания. Задача двухмассового маховика заключается в плавной, комфортной работе трансмиссии во время перехода на другую передачу. Можно сказать, что механизм состоит из двух частей, что указано даже в названии комплектующих. Один из дисков соединен с коленвалом, второй закреплен на сцеплении. В одно целое двухмассовый маховик соединяют подшипники, благодаря которым становится возможным осевое движение комплектующих относительно друг друга. Между дисками размещена сложна демпферная система. В результате трансмиссия переходит в разные режимы работы незаметно для водителя, колебания не передаются на кузов. Недостатком механизма можно с полным основанием считать чрезмерно высокую стоимость.

Неисправности сцепления

Человек за рулем в состоянии самостоятельно обнаружить поломки, возникающие в трансмиссии автомобиля. Характерные признаки неполадок сцепления могут проявляться следующим образом:

  • Наблюдается буксование деталей, что говорит о критическом износе расходника, именуемого ведомым диском. Возможны также проблемы с маховиком, гидравликой, вилкой.
  • В том случае, когда сцепление «ведет», наблюдаются сложности с расхождением дисков. Возможно произошла деформация диска, накладок, причина может крыться в диафрагменной пружине.
  • Случаются и рывки автомобиля при работе сцепления. Кроме дисков, пружин надо обязательно проверить состояние опор мотора. Кстати, сильно изношенные опоры вполне могут привести к возникновению вибрации при работе сцепления.
  • Характерный гул при выжатой педали свидетельствует о необходимости замены выжимного подшипника.
  • Если сцепление банально невозможно выключить, проблема может крыться в тросе, в гидравлике либо в электронике – в зависимости от принципа действия механизма.

Эксплуатация сцепления

Есть десятки факторов, способствующих ускоренному выходу из строя сложного механизма. Мы постараемся перечислить основные моменты, которые должен знать водитель:

  • Прежде всего, это манера вождения. Стремительные ускорения, резкие старты автомобиля негативно отражаются на ресурсе сцепления. Нельзя резко бросать педаль, таким образом вы просто «сжигаете» сцепление.
  • Сокращают запас прочности частые, продолжительные попытки выбраться на застрявшем автомобиле враскачку.
  • Нельзя перегружать машину сверх допустимого производителем предела, буксировать тяжелые прицепы.
  • Немало зависит от качества установленного комплекта.
  • Наконец, монтаж сцепления стоит доверять только компетентным специалистам на заслуживающем доверия СТО. В таком случае, можно рассчитывать на безупречный результат.

Основы устройства и принцип действия сцепления

Основы устройства и принцип действия сцепления

На автомобилях получили применение сухие дисковые сцепления.

Действие такого сцепления основано на использовании сил трения, возникающих между трущимися поверхностями. Простейшее дисковое сцепление имеет ведомый диск, установленный ступицей на шлицах конца вала коробки передач и расположенный между маховиком и нажимным диском, соединенным с маховиком через кожух, прикрепленный к маховику.

имной и ведомый диски прижимаются к маховику в сцеплении при помощи нажимных пружин, установленных в кожухе. Разъединение ведомой части сцепления от ведущей осуществляется механизмом выключения, управляемым ножной педалью.

Сцепления, трение у которых между дисками происходит при сухих трущихся поверхностях, называются сухими.

По числу ведомых дисков сцепления бывают однодисковые и двухдисковые, по устройству нажимного механизма — простые и полуцентробежные, по числу нажимных пружин сцепления — с несколькими пружинами, расположенными равномерно по окружности (периферийными) и с одной центральной пружиной.

Механизм выключения сцепления может быть с механическим или гидравлическим приводом от педали. При установке гидравлического привода повышается удобство управления сцепдением и увеличивается плавность его включения, а также улучшается герметизация пассажирского помещения кузова и размещение в нем педали сцепления, которая обычно делается подвесной. В некоторых сцеплениях для снижения усилия на педали в приводе применяют специальные усилители, обычно пневматического типа.

Чистота выключения сцепления достигается принудительным отведением нажимного диска от маховика на определенное расстояние с помощью рычагов выключения или специальных пружин при нажатии на педаль сцепления.

Надежность работы сцепления под нагрузкой без пробуксовывания дисков во включенном состоянии обеспечивается достаточной силой трения между дисками, создаваемой соответствующей силой нажимных пружин. Для того чтобы увеличить трение между дисками, поверхность ведомого диска в сухом сцеплении облицовывают с обеих сторон накладками из прессованного асбеста или медно-асбестовой плетенки.

Рис. 1. Схема работы дискового сцепления

Плавность включения сцепления получается в результате постепенного отпускания педали при включении, а также применения пружинящего ведомого диска.

Пружинение ведомого диска обеспечивается различными способами, например тем, что иедомый диск разделен на отдельные секции, которые имеют изгиб поочередно в разные стороны. Одна накладка прикреплена к секциям с изгибами вперед, а вторая — к секциям с изгибом назад, поэтому в свободном состоянии между накладками имеется зазор 1 — 2 мм. Иногда под одну из накладок устанавливают плоские пружины.

Рис. 2. Детали ведомого диска сцепления и схемы работы гасителя крутильных колебаний

При включении сцепления ведомый диск сжимается, вследствие чего трущиеся поверхности плавно соприкасаются и сила трения между ними постепенно возрастает.

На ведомом диске сцепления обычно ставят гаситель крутильных колебаний. Гаситель предохраняет силовую передачу от возникновения на ее валах значительных крутильных колебашш, вызывающих повышение нагрузок на детали. Эти колебания и рост напряжений в деталях могут возникать из-за неравномерности вращения коленчатого вала или его крутильных колебаний, а также в результате резких изменений угловых скоростей на валах в силовой передаче при движении автомобиля по неровным дорогам. Гаситель, кроме того, обеспечивает большую плавность включения сцепления. Все это снижает нагрузки, действующие на детали силовой передачи, способствуя повышению их долговечности.

При наличии гасителя ведомый диск сцепления соединяют со ступицей не жестко, а при помощи шести-восьми пружин. Пружины установлены в сжатом состоянии в прямоугольных вырезах фланца ступицы, ведомого диска и диска гасителя. Диски соединены штифтами. Для увеличения трения между фланцем ступицы и дисками между ними иногда устанавливают фрикционные кольца из специального картона (паронита). Сила сжатия между дисками регулируется подбором регулировочных стальных колец.

В свободном состоянии, когда усилие через диск не передается, прорези диска и фланца ступицы совпадают.

При включении сцепления усилие от диска на ступицу передается через пружины. Под действием этого усилия пружины сжимаются, диск несколько смещается относительно ступицы, и плавность включения сцепления увеличивается.

Крутильные колебания, возникающие на валах, вызывают угловые смещения ведомого диска относительно его ступицы вследствие деформации пружин, что сопровождается трением между дисками в гасителе и гашением колебаний.

Предохраняющее значение сцепления заключается в том, что в случае резкого замедления коленчатого вала двигателя (например, при торможении с невыключенным сцеплением) возникающий на нем значительный по своей величине инерционный момент вызывает пробуксовывание дисков сцепления п не передается полностью на силовую передачу. Это предохраняет силовую передачу от поломок.

Как устроено сцепление автомобиля, принцип действия и виды

Автомобиль состоит из множества сложных узлов и механизмов. Каждый элемент играет свою незаменимую роль. Если исключить сцепление из общей цепочки, автомобиль будет трогаться с места рывками, а двигатель подвергаться большим нагрузкам. Коробка передач в таких условиях эксплуатации прослужит не более трех дней.

Содержание

  • Сцепление: общие сведения и назначение, функции
  • Устройство и составляющие сцепления
  • Принцип работы и механизм
  • Принцип работы приводов
  • Виды сцепления и классификация
  • Особенности сцепления АКПП
  • Характеристики керамического и металлокерамического сцепления

Сцепление: общие сведения и назначение, функции

Сцепление является неотъемлемой частью трансмиссии, а располагается между двигателем и КПП автомобиля, обеспечивая ступенчатое переключение передач, контроль крутящего момента и временное прерывание связи маховика и трансмиссии.

Принцип работы сцепления основывается на силе трения, а если точнее – скольжения. Состоит система сцепления из привода и непосредственного механизма.

При необходимости резкого торможения именно сцепление может уберечь узел от перегрузки.

Управление в автомобилях с механической коробкой передач происходит за счет педали сцепления. С ее помощью удается соединять и разрывать связь между двигателем и КПП. Если педаль отпустить резко, пружина стремительно вернет ее в исходную позицию.

Езда на транспортном средстве с механической коробкой передач при постоянно выжатом сцеплении спровоцирует перегрев и быстрый износ элементов. Езда с пробуксовкой допустима в экстремальных условиях, для поднятия оборотов.

В стандартном виде сцепление отсутствует в гидромеханических КПП и вариаторах. Хотя, в гидромеханических коробках используются фрикционные муфты для плавного переключения передач. Встретить классическую сборку возможно лишь на РКПП, где процессом переключения управляют сервоприводы (гидравлические или электронные). Очень часто в РКПП используются два сцепления для оптимизации процесса и устранения задержек переключения – когда одно сцепление работает, другое в состоянии ожидания для переключения следующей передачи.

Устройство и составляющие сцепления

Устройство сцепления условно можно разделить на две части: механизм и привод. В целом в конструкцию узла входит:

  1. Нажимной диск или корзина. Является основой для других конструктивных элементов сцепления. Имеет непосредственный контакт с выжимными пружинами, которые направлены к центру. Размер площадки пропорционален двум радиусам маховика ДВС. Прижимной участок отличается наличием шлифовки исключительно с одной стороны. Диск имеет плотное соединение с маховиком двигателя.
  2. Ведомый диск. Располагается в зазоре прижимного участка и маховика. Имеет непосредственный контакт с КПП при помощи шлицевой муфты и фрикционных накладок. Вокруг муфты конструктивно находятся демпферные пружины, которые принимают на себя всю вибрацию.
  3. Фрикционные накладки. Находятся в основании и изготавливаются из различных композитных материалов.
  4. Выжимной подшипник. Визуально делится на две части, одна из которых имеет круглую основу для воздействия на пружины корзины. Подшипник расположен на кожухе вала. Существует два типа подшипников: оттягивающего или нажимного принципа. Первый тип нашел свое применение в Peugeot. Иногда подшипник имеет несколько пружин-фиксаторов.
  5. Привод и педаль сцепления. В автоматических коробках сохранен только механизм.

Принцип работы и механизм

Вся работа сцепления построена на трении между дисками. Ведущий диск является частью ДВС, а ведомый диск – элемент трансмиссии. Когда водитель отпускает педаль, то пружины сжимают диски вместе. В итоге за счет фрикционных поверхностей, диски притираются и продолжают вращение с равной угловой скоростью. От силы лепестков пружин зависит показатель абразива диска.

Когда водитель выжимает сцепление, основа привода перемещают вилку, которая впоследствии оказывает влияние на подшипник. Последний перемещается до упора. Пружины в этот момент уже готовы прижать два диска, что значит, что вилка разорвала связь между трансмиссией и маховиком ДВС. Все трансмиссионные удары, когда водитель резко бросает педаль, когда ТС тронулось с места, поглощают и сглаживает отдельный тип пружин.

Принцип работы приводов

Привод напрямую влияет на исправность всего узла и необходим для дистанционного управления из салона. В общей системе выделяют три основных типа:

  • Механический привод сцепления. Является одним из самых распространенных. Усилие передается при помощи троса к вилке. Конструкция находится под покрытием кожуха, который находится перед педалью и вилкой.
  • Гидравлический. Предполагает наличие основного и рабочего цилиндра, которые связаны под большим давлением трубками. После того как водитель нажимает на педаль, активируется шток. Действующий в итоге поршень имеет стойкую манжету и передает давление жидкости к рабочему цилиндру. Последний имеет отдельный шток, который давит на вилку. Используемая в системе жидкость размещается в отдельном бачке.
  • Электрический привод. По принципу действия схожий с механическим приводом. Единственное отличие заключается в срабатывании мотора при давлении на педаль.

Нажатие на педаль сцепления позволяет напрямую оказывать воздействие на нажимной диск автомобиля.

Виды сцепления и классификация

Сегодня автомобилисты выделяют множество классификаций сцепления. Можно встретить однодисковые или многодисковые механизмы. Кроме того, сцепление бывает сухими и мокрым, на это влияет среда, в которой работает узел. Самое большое распространение имеет сухое однодисковое сцепление. Отдельную классификацию выделяют относительно типа рабочего привода и относительно принципа нажатия на корзину.

По характеру силы трения существует два вида: сухое и мокрое. Сухое – обеспечивается за счет функциональной работы передачи вращения между двумя шкивами. Мокрое сцепление работает за счет передачи энергии при помощи сжатия компонентов, находящихся в автомобильном масле.

Отдельно существует различие по количеству шкивов:

  • Однодисковые. Системы, которые характерны как для легкового транспорта, так и для грузового. Элемент применим для автомобилей, у которых крутящий момент попадает в диапазон 0,7–0,8 кНм.
  • Многодисковая система. Применима для тяжелых транспортных средств с высоким крутящим моментом. В конструкции предусмотрено наличие двух рабочих дисков, корзины и системы контроля синхронного нажатия.

Если рассуждать относительно расположения пружин на дисках, то можно отметить, что встречаются два варианта: демпферные пружины помещены по периферии и наличие централизованной диафрагмы.

Особенности сцепления АКПП

Чаще всего автомобили с автоматической коробкой наделенны влажным многодисковым типом сцепления, хотя можно встретить варианты сухого сцепления. Управление выжимной силой, как и переключение передач, происходит за счет работы сервопривода. Актуаторы бывают гидравлические и электрические. Управление сервоприводами происходит при помощи ЭБУ или гидрораспределителя.

Больше всего негодований вызывает работа электрических сервоприводов во время переключения передач. Прежде чем, запустить в работу механизм сцепления, акутатор проводит анализ оборотов двигателя и только потом разъединяет ДВС от трансмиссии. Гидравлический сервопривод реагирует на давление, созданное распределителем и масляным насосом при достижении определенного показателя оборотов. После чего запускает в ход механизм сцепления.

Характеристики керамического и металлокерамического сцепления

В последнее время любители экстремальной быстрой езды открыли для себя керамическое и металлокерамическое сцепление. Керамика значительно выигрывает, если ее установить на мощный агрегат, который любит стартовать с пробуксовкой и сжигать резину. Металлокерамическое сцепление может выдерживать значительные нагрузки и является лучшим выбором гонщиков.

Диски производят с добавление углеродистого волокна, кевлара и керамики. Такой состав позволяет на 10–15% поднять передачу крутящего момента без увеличения прижимной силы, оказываемой на корзину. Живут такие диски, как правило, в четыре раза дольше обычных. Производят 3-х, 4-х, 6-и лепестковые модели, которые отлично справляются с температурными и механическими нагрузками. Некоторые водители жалуются на слишком резкое переключение передач при керамическом сцеплении, но определенного
мнения на этот счет среди автомобилистов пока нет.

Чтобы детально понимать принцип работы сцепления автомобиля теорию необходимо подкреплять практикой. Если такой возможности нет, увидеть наглядный пример можно на роликах в сети:

Устройство и принцип работы сцепления, как работает сцепление, диск сцепления, признаки неисправности. Устройство и принцип работы сцепления. Как работает сцепление. Его устройство и основные неисправности.

Сцепление автомобиля — одно из самых основных конструктивных составляющих транспортного средства. Его главное предназначение — кратковременное отсоединение мотора от трансмиссии и плавное их соединение друг с другом при переключении передач. Автомобильное сцепление также защищает составляющие трансмиссии от перегрузок. Данный конструктивный элемент расположен между мотором и КПП.

Содержание

  • Устройство сцепления, принцип работы сцепления автомобиля
  • Какое сцепление на авто, классификация сцеплений для легковых и грузовых машин
  • Неисправности сцепления, признаки неисправности сцепления
  • Советы профи, как предупредить неисправность сцепления

Устройство сцепления, принцип работы сцепления автомобиля

Элементы сцепления:

  1. Первичный вал КПП.
  2. Маховик мотора.
  3. Вилка выключения сцепления.
  4. Картер КПП.
  5. Отжимные рычаги.
  6. Шестерня КПП.
  7. Выжимной подшипник.
  8. Нажимной диск.
  9. Пружины.
  10. Кожух сцепления.
  11. Картер сцепления.

Чтобы понять предназначение сцепления, следует сопоставить работу мотора с таким понятием, как «движение транспорта». Если представить себе, что маховик двигателя непосредственно соединен с ведущим мостом автомашины, то при запуске мотора, автомобиль должен сразу поехать. Соответственно, чтобы остановить машину, нужно заглушить мотор. Именно для этого и необходимо сцепление, позволяющее в подходящий момент получать от мотора энергию в начале движения либо прерывать данный процесс для прекращения езды.

Классическое сцепление состоит из ведомого и нажимного дисков, а также привода, который заставляет их прижиматься либо разъединяться друг с другом. Закреплена данная конструкция в кожухе, который твердо крепится к маховику коленвала. Нажимной диск довольно массивный и также твердо крепится в кожухе. Стоит отметить, что ведомый диск гораздо тоньше и находится на шлицах основного (первичного) вала КПП автомобиля. Шлицы отвечают за обеспечение его подвижности вдоль оси вала, а также за жесткую сцепку с валом. Что касается нажимного диска, то он не имеет такой сцепки с валом КПП.

В стандартном рабочем положении нажимной и ведомый диски прижаты друг к другу с помощью мощных пружин посредством рычагов и выжимного подшипника. Следовательно, в результате силы трения между данными дисками, на первичный вал КПП от маховика передается крутящий момент. Если отвести нажимной диск от ведомого, происходит прерывание крутящего момента от мотора и прекращение вращения ведомого диска с валом.

Отсоединение дисков производится с помощью вилки сцепления, которая своим видом напоминает детские качели. Сама вилка приводится в действие посредством цепочки рычагов и тяг педалью сцепления, которая расположена в кабине.

Перед запуском мотора автомобилист нажимает на педаль сцепления, воздействуя на вилку посредством тяги и вынуждая ее противоположные концы перемещаться относительно центра в противоположные стороны. Конец вилки оказывает давление на выжимной подшипник, который посредством рычагов заставляет сжиматься пружины, оказывающие давление на нажимной диск. Последний отделяется от ведомого диска, что прерывает передачу вращающего момента. В результате при выжатой педали сцепления и запущенном моторе вращается только маховик. Чтобы начать движение, необходимо плавно отпустить сцепление, из-за чего по данной цепочке вилка прекратит оказывать свое воздействие на нажимной подшипник, что, в свою очередь, смягчит давление на рычаги. Затем пружины начнут разжиматься, в результате чего придавят к маховику ведомый и нажимной диски. Поскольку ведомый диск твердо крепится к шлицам первичного вала КПП, крутящий момент от мотора будет передаваться по трансмиссии ведущим колесам и транспортное средство начнет движение.

Необходимо отметить, что существует два типа привода сцепления — гидравлический и механический. Механический вариант является самым простым в работе сцепления авто. В данном случае, автомобилист, нажимая на педаль, оказывает влияние на вилку сцепления посредством тяг и тросов. Гидравлический вариант предусматривает поршень с жидкостью. Чаще всего его применяют на большегрузном транспорте, чтобы облегчить работу водителя.

Какое сцепление на авто, классификация сцеплений для легковых и грузовых машин

Зависимо от конструкции сцепление бывает таких типов — электромагнитное, фрикционное и гидравлическое.

Фрикционный вариант сцепления выполняет передачу вращающего момента с помощью силы трения. Сцепление электромагнитного вида контролируется с помощью магнитного поля. В гидравлическом варианте сцепления связь обеспечивается при помощи потока жидкости.

Фрикционный тип сцепления является наиболее распространенным. Зависимо от количества дисков различают такие виды фрикционного сцепления — многодисковые, однодисковые и двухдисковые.

Сцепление бывает мокрое и сухое. В сухом сцеплении предполагается работа дисков в условиях сухого трения. Мокрое сцепление предусматривает эксплуатацию дисков в жидкости.

Как правило, современные автомобили оснащены сухим однодисковым сцеплением. Все компоненты сцепления расположены в картере, который при помощи болтов крепится к двигателю.

Гидравлическое сцепление. Гидромуфта, где передача крутящегося  момента осуществляется гидродинамическим напором жидкости, которая циркулирует между ведущими и ведомыми компонентами, называется гидравлическим сцеплением.

Гидромуфта не применяется на автомобилях в качестве независимого сцепления, поскольку не способна обеспечить абсолютного выключения, что существенно усложняет переключение передач. В результате этого при применении гидромуфты вместе с ней устанавливается фрикционное сцепление, которое предназначено только для переключения передач. При этом во фрикционном сцеплении монтируются более мягкие и пластичные нажимные пружины, что облегчает выключение сцепления.

Электромагнитное сцепление. Сцепление считается электромагнитным, если сжатие ведущих и ведомых элементов осуществляется посредством электромагнитных сил. Электромагнитное сцепление постоянно находится в разомкнутом состоянии.

Грузовые и легковые автомобили с мощным мотором оснащены двухдисковым сцеплением, которое при неизменном размере осуществляет передачу существенно большего крутящего момента, а также предоставляет значительно больший ресурс конструкции. Это достигается за счет применения двух ведомых дисков, посреди которых находится проставка. В результате получены четыре поверхности трения.

Неисправности сцепления, признаки неисправности сцепления

  1. Сцепление не полностью включается («пробуксовывает») в результате замасливания или износа фрикционных накладок ведомого диска, поломки пружин, неудовлетворительной амплитуды хода педали. Для устранения данной неисправности, замените ведомый диск, устраните задиры на дисках, смените неисправные узлы привода.
  2. Сцепление не полностью выключается («ведет») в результате довольно большого свободного хода сцепления, поломки пружины, коробления ведомого диска или несоответственно стоящего нажимного диска.  Для устранения данной неисправности сцепления, следует удалить из гидропривода воздух, отрегулировать свободный ход педали, произвести замену неработоспособных дисков и пружин.
  3. В приводе выключения сцепления подтекает тормозная жидкость, что возможно из главного и рабочего цилиндров, а также в соединительных трубках. Чтобы устранить неисправность, визуально найдите место утечки и поменяйте на новые неисправные узлы, после чего прокачайте гидропривод полностью.

Советы профи, как предупредить неисправность сцепления

Чтобы предупредить рассмотренные выше неисправности сцепления, достаточно придерживаться простых правил эксплуатации. При эксплуатации сцепления необходимо периодически осуществлять проверку уровня тормозной жидкости в бачке. Если уровень ниже нормы, обязательно восстановите его.

Пониженный уровень тормозной жидкости или неправильная регулировка сцепления способна привести к тому, что передачи на вашем авто будут тяжело включаться или перестанут включаться вообще. При движении водителю приходится постоянно выжимать и отпускать педаль сцепления, что вынуждает поверхности ведомого диска с большой силой тереться о нажимной диск и маховик, в результате чего, разумеется, со временем боковые поверхности ведомого диска изнашиваются. Это обыкновенный процесс, который предусмотрен конструкцией автомобиля. Любая машина требует внимания к себе. В среднем, при правильной эксплуатации сцепления, замена ведомого диска нужна после 80 000 километров езды.

Устройства сцепления трактора | принцип работы сцепления трактора

Меню

  • Новости
  • Статьи
  • Видеоматериалы
  • Фотоматериалы
  • Публикация в СМИ
  • 3D-тур

Будь в курсе

Новости, обзоры и акции

18. 07.2020

Тракторы функционируют за счет установки фрикционных сцеплений. Последние для осуществления передачи крутящего момента используют с физической точки зрения силы трения, которые возникают при взаимодействии ряда деталей конструкции.

Особенности главного сцепления трактора

Сцепление предназначено для решения ряда задач, поэтому может иметь разные конструктивные особенности. Перечислим:

  • для трогания с места в плавном режиме;
  • разъединения соединения между двигателем и трансмиссии во время переключения передач;
  • предохранения трансмиссионной передачи при использовании разных режимов работы трактора.

Вся спецтехника делится на подвиды, которые снабжены одним или сразу несколькими сцеплениями. Главным называется сцепление, расположенное сразу за дизелем. Входит в конструкцию трансмиссии. Принцип действия базируется на опирании дизеля на коленчатый вал. Это производится через шарикоподшипник, который запрессован на торцевой части.

Если ведомый диск поставить рядом с маховиком, то вращение производиться не будет. Но при перемещении его в левую сторону и увеличении прижимания по направлению к маховику произойдет усиление сил трения, за счет чего увеличится прижатие диска. Все это в сумме и составляет принцип работы сцепления у трактора, так как организует плавное соединение валов.

Механизмы для главного сцепления

Для обеспечения правильного принципа работы сцепления у трактора применяются механизмы с различными конструктивными особенностями. Но при этом устройство сохраняется на основании вышеописанной схемы действия.

Сцепление трактора содержит основу, которой является специальный кожух. Внутри него размещены ведущий диск и пружины нажимного формата. Кожух закрепляется на поверхности маховика. Между последним и ведущим диском производят установку ведомого, который выполнен из тонкого стального профиля. С двух сторон последнего располагаются асбестовые накладки, в состав которых добавлены материалы для увеличения эффекта связывания.

Накладки в устройстве сцепления отвечают за стойкость по отношению к высоким температурным воздействиям, которые появляются вследствие пробуксовки диска при процессах включения и выключения. При этом они имеют высокий коэффициент трения.

Ступица ведомого диска расположена на валовых шлицах. Сам диск прижимается к маховику за счет пружин, которые размещены с плотным упором по отношению к кожуху. Последние вставлены внутрь специальных стаканов. Реализация плотного эффекта зажимания вызывает силы трения. Вращение маховика дает крутящий момент, за счет чего трение передается ведомому валу и переходит затем к ведущему.

Такой принцип действия сцепления работы трактора является фрикционным и соответствует варианту с использованием одного диска. За счет этого цепь замкнута. Но возможен и формат с использованием двух ведомых дисков.

Для этого на спецтехнику устанавливают дизель с большой мощностью. На него передаются сильные крутящие моменты, которых достичь одна единица не может из-за меньшей прочности. Такое фрикционное сцепление будет двухдисковым и тоже замкнутым.

Механизм управления всем устройством сцепления заключается в соединении рычагов, вилки и педали. Система включает передвижную муфту и шарикоподшипник упорного типа с выключающими рычагами. Помимо этого она связана напрямую с ведущим диском.

Выключение сцепления производится нажатием на педаль. Это сопровождается перемещением муфты в левую сторону, что вызывает автоматическое нажимание рычагов. Поворот деталей на осях отведет ведущий диск в нужном направлении. Таким образом происходит прекращение передачи крутящего момента.

Для включения сцепления трактора педаль вызывает действие пружин, что помогает снова выполнить маховику нужное перемещение и прижать ведомый диск.

Пробуксовка дисков вызывает повышение температурного режима, поэтому медленное включение сцепления не рекомендовано для применения, так как способно вызвать перегрев и возникновение неисправностей. Ускоренное или замедленное включение не соответствует нормативам использования машины, которое равно 1,5-3 сек.

Дополнительные механизмы

Такой формат устройств используется для получения улучшенных показателей. Механизмы отвечают за уменьшения усилия, затрачиваемого на включение, позволяют погасить колебания в момент кручения и ускорить своевременное выключение ведомого диска.

  1. Для уменьшения усилия. Принцип выключения сцепления трактора базируется на действии ряда пружин, обладающих высокой упругостью. Воздействие на них требует значительных усилий. Чтобы выполнить задачу, производят установку усилителей разного типа: на основе механики, гидравлики или пневматики.
  2. Гашение колебаний кручения. Такие процессы вызваны при работе коленчатого вала, который претерпевает многочисленные раскручивания и закручивания вплоть до определенных значений деформации. Для предупреждения износа в будущем и увеличения срока эксплуатации соединение между дисками и ступицей выполняется при помощи установки дополнительных элементов. Ими служат демпферы на основе резины, либо пружинные механизмы.
  3. Включение передач у сцепления затруднено из-за присутствия инерционных процессов. Чтобы избежать ее увеличения и погасить, используется установка тормозка.

Фрикционное сцепление

Такой вид сцепления приобрел в современно мире наибольшую популярность. Связано это с оптимальным соотношением цены и качественных показателей. Уменьшенные габариты и высокая степень надежность являются их основными характеристиками.

Виды фрикционных сцеплений делятся на подвиды в зависимости от ряда конструктивных особенностей:

  • Рабочие поверхности могут перемещаться в двух направлениях, поэтому изделия подразделяются на радиальные и осевые.
  • Форма этих поверхностей также может различаться, поэтому бывают дисковые, колодочные, ленточные и конусные. Первые обладают большей степенью надежности, поэтому применяются чаще всего.
  • Использование одного, двух и более дисков также реализует разные варианты действия сцеплений.
  • Сухие способны работать без смазывающих веществ, а мокрые взаимодействуют с поверхностями при помощи масляной основы.
  • В зависимости от количества потоков мощности происходит деление на однопоточные и двухпоточные.
  • Число фрикционных механизмов влияет на передачу крутящего момента к ведущим частям, в частности, к диску.

Ведомые диски

Выполняются из основания на основе высокопрочной стали в виде колец. По двум его сторонам устанавливаются накладки, которые крепятся при помощи прочных заклепок. Для требуемого прилегания деталей организуются прорези радиального типа, которые в конце завершаются наиболее крупным диаметральным размером.

Но чаще используется не жесткий ведомый диск в сцеплении устройства трактора, а обладающий тангенциальной или осевой податливостью. Это дает возможность формирования плавности во время включения и упрощает управление машиной.

Фрикционные детали сцепления тракторов

Накладки предназначены для предупреждения износа, поэтому способны выдерживать серьезные тепловые и динамические нагрузки. Выполняются на основе полимеров или порошкообразных соединений.

В первом случае представляют собой композицию из множества компонентов. При этом состоят из основы, арматуры и наполнителя. В качестве базы используют такие вещества как каучук и смолы, а также их многочисленные комбинации. Наполнители делятся на металлические и неметаллические виды.

Размеры накладок в устройстве сцепления трактора нормируются и соответствуют ГОСТ 1786. Изготавливаются в виде колец или усеченных секторов.

Ведущие диски

Позволяют обеспечить корректное распределение тепловых потоков, так как отвечают за действие всех последующих конструктивных элементов. То есть отвечают за рассеивание и поглощение. Для обеспечения требуемых значений используются материалы в виде серого чугуна разных марок: 18, 21, 22 и 24. Они обладают высокой степенью износостойкости, уменьшают износ используемых в системе накладок.

Окружность ведомых дисков для выполнения работ во взаимосвязи с другими деталями имеет множество выступов, пальцев, шипов, зубьев, соединений из шпона и тангенциальных пружин. Все элементы равномерно распределяются по поверхности изделий.


Другие статьи

Смотреть

ещё

Двигатель ЗИЛ 130 характеристики

19.08.2020 16:09:00

Заправочные объемы ЗИЛ 130

19.08.2020 10:10:00

Cмазка компрессора ЗИЛ 130

18.08.2020 16:09:00

Воздушная система ЗИЛ 130

17.08.2020 15:10:00

Как проверить масло TOTAL на подлинность

13.08.2020 12:42:00

Трактор Т 25 замена масла

13.08.2020 12:09:00

Схема бесконтактного зажигания ЗИЛ 130

13.08.2020 10:29:00

Гидравлическое рулевое управление трактора

12.08.2020 17:24:00

Зазор свечей зажигания ЗИЛ 130

21.07.2020 16:13:00

Замена сцепления на тракторе Т 25

20. 07.2020 18:04:00

Карбюратор ЗИЛ 131

18.07.2020

Установка двигателя на трактор

15.07.2020

Норма расхода тракторов

15.07.2020

Двигатель ЗИЛ 645

14.07.2020

Двигатели ЗИЛ: модели

14.07.2020

Какое масло Тотал лучшее

13.07.2020

Какое масло заливается в трактор

10.07.2020

Производительность компрессора ЗИЛ 130

29.06.2020

Расход топлива ЗИЛ 130

29.06.2020

Расход топлива ЗИЛ 131

29.06.2020

Смотреть

ещё

Возврат к списку

Clutch Principles

Несоблюдение надлежащих мер безопасности при работе с Clutch Systems может привести к серьезным травмам\проблемам со здоровьем, например. Проблемы с дыханием у персонала.
Даны инструкции по надлежащим процедурам безопасности, применимым к работе с системами сцепления, включая безопасное использование:

  • автоподъемников,
  • Опорные балки двигателя,
  • Домкраты коробки передач,
  • Использование подходящей защиты для глаз,
  • Латексные перчатки,
  • Защитная обувь
  • Безопасное удаление пыли сцепления,
  • Использование подходящей маски для лица во избежание проблем с дыханием,
  • Работа с соответствующими инструментами сцепления,
  • Предотвращение утечки жидкости сцепления,
  • Помощь при снятии и установке коробки передач с использованием рекомендованных отраслевых методов ручного обращения и т. д.

См. оценки рисков, связанных с двигателем, экологическую политику и паспорта безопасности материалов (MSDS). Автомобильное сцепление передает крутящий момент от двигателя к трансмиссии, а водитель использует расцепляющий механизм для управления потоком крутящего момента между ними.


В большинстве легковых автомобилей используется однодисковый фрикционный диск с двумя фрикционными накладками, прикрепленными к центральной ступице со шлицами для приема входного вала трансмиссии.
Фрикционные накладки зажаты между плоской поверхностью маховика двигателя и подпружиненной нажимной пластиной, прикрученной болтами к ее внешнему краю.

3.2 Однодисковые муфты сцепления

В большинстве легковых автомобилей для передачи крутящего момента от двигателя к первичному валу коробки передач используется однодисковая муфта. Маховик является приводным элементом сцепления. Блок сцепления установлен на обработанной задней поверхности маховика, так что блок вращается вместе с маховиком. Блок сцепления состоит из фрикционного диска с двумя фрикционными накладками и центральной шлицевой ступицы.
Узел нажимной пластины, состоящий из штампованной стальной крышки, нажимной пластины с обработанной плоской поверхностью, сегментированной диафрагменной пружины, выжимного подшипника и рабочей вилки.
Фрикционный диск зажат между обработанными поверхностями маховика и прижимной пластины, когда прижимная пластина прикручена болтами к внешнему краю поверхности маховика.

Прижимное усилие на фрикционных накладках обеспечивается диафрагменной пружиной. В разгруженном состоянии он имеет выпуклую форму. Когда крышка прижимной пластины затягивается, она поворачивается на своих опорных кольцах и расплющивается, оказывая усилие на прижимную пластину и облицовку.
Входной вал трансмиссии проходит через центр нажимного диска. Его параллельные шлицы входят в зацепление с внутренними шлицами центральной втулки на фрикционном диске.
При вращении двигателя крутящий момент теперь может передаваться от маховика через фрикционный диск к центральной втулке и к трансмиссии. Группа торсионных пружин, расположенных между ступицей сцепления и накладкой, гасит удары и вибрации трансмиссии.


При нажатии педали сцепления движение передается через рабочий механизм на рабочую вилку и выжимной подшипник.
Выжимной подшипник перемещается вперед и толкает центр диафрагменной пружины к маховику.
Мембрана поворачивается на своих кольцах шарнира, заставляя внешний край двигаться в противоположном направлении и воздействовать на зажимы втягивания прижимной пластины. Прижимная пластина отключается, и привод больше не передается. Отпускание педали позволяет диафрагме повторно приложить усилие зажима и включить сцепление, и привод восстанавливается.

3.3 Прижимная пластина

В легковых автомобилях нажимная пластина обычно представляет собой мембранный тип и обслуживается как узел.

Он состоит из штампованной стальной крышки, прижимной пластины с обработанной плоской поверхностью, ряда приводных ремней из пружинной стали и диафрагменной пружины.
Эта диафрагма расположена внутри крышки сцепления на 2 опорных кольцах, удерживаемых рядом заклепок, проходящих через диафрагму.
Нажимная плита соединена с крышкой приводными ремнями из пружинной стали, приклепками к крышке с одного конца и к выступающим выступам на плите — с другой.
Втягивающие зажимы удерживают прижимную пластину в контакте с внешним краем диафрагмы. Во время работы сцепления они отодвигают диск от маховика.


3.4 Ведомая / центральная пластина

Ведомая центральная пластина также называется диском сцепления или фрикционным диском.

Ведомая пластина имеет пару фрикционных накладок из армированного проволокой безасбестового состава, закрепленных на волнистых сегментах из пружинной стали, приклепываемых к стальному диску.
Центральная шлицевая втулка из легированной стали является отдельной. Привод передается от диска к ступице через тяжелые винтовые пружины кручения или резиновые блоки. Такая пружинная ступица гасит крутильные колебания двигателя. Он также поглощает ударные нагрузки, воздействующие на трансмиссию при внезапном или резком включении сцепления.
Стопоры ограничивают радиальное перемещение ступицы против усилия пружины. Формованная фрикционная шайба между ступицей и удерживающей пластиной пружины также действует как демпфер.
Волнистые сегменты из пружинной стали заставляют накладки немного расширяться, когда сцепление выключено, а затем сжиматься при включении. Это имеет амортизирующий эффект и обеспечивает плавное зацепление.


3.5 Подшипник выключения сцепления (выжимной подшипник)

Подшипник выключения сцепления может быть упорным радиально-упорным шарикоподшипником, поддерживаемым на водиле. Он скользит по ступице или втулке, выступающей из передней части трансмиссии.

Держатель подшипника находится на вилке выключения сцепления. При перемещении вилки упорная поверхность подшипника соприкасается с пальцами нажимного диска. Это заставляет подшипник вращаться и поглощать вращательное движение пальцев против линейного движения вилки. Подшипник заполнен смазкой при изготовлении и не требует периодического обслуживания в течение всего срока службы.

3.6   Двухмассовые маховики

В современных легкодизельных технологиях мы наблюдаем гораздо большую мощность и прирост крутящего момента, иногда в сочетании с большей экономией топлива.


Преимущества двухмассовых маховиков

Чтобы устранить чрезмерное дребезжание шестерен трансмиссии и сделать вождение комфортным на любой скорости, уменьшите усилие при переключении передач.

Зачем нужен двухмассовый маховик?

Трансмиссии легких грузовиков Автомобили с дизельным двигателем по умолчанию имеют повышенную чувствительность к колебаниям крутящего момента. Это приводит к сильному крутильному резонансу или вибрации, возникающим при эксплуатации автомобиля в пределах нормального диапазона движения.
Благодаря гашению вибраций, превосходящему нормальное демпфирование в обычном механизме сцепления, автомобиль может эксплуатироваться в течение более длительного времени без долговременных повреждений.
Конструкция двухмассового маховика перемещает демпфер с ведомого диска на маховик двигателя. Такое изменение положения гасит крутильные колебания двигателя в большей степени, чем это возможно при использовании стандартной технологии демпфирования диска сцепления.

Назначение и работа

Функция двухмассовых маховиков или двухмассовых маховиков состоит в том, чтобы изолировать торсионные шипы коленчатого вала, создаваемые дизельными двигателями с высокой степенью сжатия. Устраняя торсионные шипы, система устраняет любое потенциальное повреждение зубьев шестерни трансмиссии. Если бы DMF не использовался, крутильные частоты могли бы повредить трансмиссию.

3.7   Рабочие механизмы

Движение накладки педали передается через рабочий механизм на узел сцепления на задней части маховика.
Этот механизм может быть механическим или гидравлическим.
Механические системы могут использовать систему рычагов, но тросовое управление обеспечивает большую гибкость и является более распространенным.


В гидравлическом управлении сцеплением педаль воздействует на главный цилиндр, соединенный гидравлической трубой и гибким шлангом с рабочим цилиндром, установленным на картере сцепления.
Рабочий цилиндр приводит в действие вилку выключения сцепления. В гидравлических системах сцепления важно, чтобы в системе не было воздуха, так как он будет сжиматься и не позволит давлению передаваться на вилку выключения сцепления. Поэтому важно прокачать систему, и это должно быть сделано с использованием заводских процедур.

4.1   Рычаг / механическое преимущество

Механическое преимущество

В физике и технике механическое преимущество (MA) — это коэффициент, на который машина умножает прилагаемую к ней силу.

Рычаги

В физике рычаг — это жесткий объект, который используется с соответствующей точкой опоры или точкой поворота для умножения механической силы, которая может быть приложена к другому объекту. Это также называется механическим преимуществом (ma) и является одним из примеров принципа моментов.

Сила и рычаги

Приложенная сила (в конечных точках рычага) пропорциональна отношению длины плеча рычага, измеренного между точкой опоры и точкой приложения силы, приложенной к каждому концу рычага.


Три класса рычагов

Существует три класса рычагов, представляющих собой варианты расположения точки опоры и входных и выходных усилий.

Рычаги первого класса

Примеры: Рычаги первого класса

  • Качели
  • Лом (удаление гвоздей)
  • Плоскогубцы (двухрычажные)
  • Ножницы (двухрычажные)
  • Весло для гребли, рулевого управления или гребли

Рычаги второго класса

Примеры: Рычаги второго класса

  • Тачка
  • Щелкунчик (двухрычажный)
  • Лом (разъединяющий два предмета)
  • Ручка кусачек для ногтей

 

Рычаги третьего класса

Примеры: Рычаги третьего класса

  • Рука человека
  • Щипцы (двухрычажные) (с петлей на одном конце, стиль с центральной осью является первоклассным)
  • Основной корпус кусачек для ногтей, в котором рукоятка создает поступающее усилие

Моменты

Принцип моментов гласит, что когда тело находится в равновесии, то сумма моментов по часовой стрелке относительно любой точки равна сумме моментов против часовой стрелки относительно той же точки.


Гидравлическое давление и усилие

Гидравлические системы сцепления используют несжимаемую жидкость, такую ​​как тормозная жидкость, для передачи усилий из одного места в другое внутри жидкости. Большинство автомобилей также используют гидравлику в тормозных системах. Закон Паскаля гласит, что при повышении давления в любой точке замкнутой жидкости происходит такое же увеличение во всех остальных точках сосуда.

Гидравлическое давление передается через жидкость. Поскольку жидкость фактически несжимаема, давление, приложенное к жидкости, передается без потерь по всей жидкости. В тормозной системе это позволяет усилию, приложенному к педали тормоза, воздействовать на тормоза на колесах.
Гидравлическое давление может передавать повышенную силу. Поскольку давление — это сила, приходящаяся на единицу площади, одно и то же давление, прикладываемое к разным площадям, может создавать разные силы — большие и меньшие.

Давление

Давление — это приложение силы к поверхности и концентрация этой силы в данной области. Палец можно прижимать к стене, не оставляя длительного впечатления; однако тот же палец, нажимающий кнопку, может легко повредить стену, даже если приложенная сила одинакова, потому что острие концентрирует эту силу на меньшей площади.


Расчет соотношения сил (гидравлика)

В обычном гаражном домкрате плунжер диаметром 10 мм нагнетается в поршень диаметром 50 мм. Это даст соотношение сил 25:1.
Площадь плунжера     = Þr2
= 3,14 х (52)
= 78,5 мм2
Площадь Рама          = Þr2
= 3,14 х (252)
= 1962,5 мм2
Отношение сил             Площадь ползуна                1962,5 = 25
Площадь плунжера              78,5
Ф.Р = 25:1
В тормозной системе главный и рабочий цилиндры имеют такой размер, чтобы соотношение усилий составляло 4:1 (приблизительно)


4.

3 Трение

 

Обзор

Трение — это сила, которая сопротивляется движению одной поверхности по другой. В некоторых случаях это может быть желательно; но чаще не желательно. Это вызвано поверхностными шероховатыми пятнами, которые сцепляются друг с другом. Эти пятна могут быть микроскопически малы, поэтому даже кажущиеся гладкими поверхности могут испытывать трение. Трение можно уменьшить, но никогда не устранить.
Трение всегда измеряется для пар поверхностей с использованием так называемого коэффициента трения.

  • Низкий коэффициент трения для пары поверхностей означает, что они могут легко перемещаться друг по другу.
  • Высокий коэффициент трения для пары поверхностей означает, что они не могут легко перемещаться друг по другу.

Коэффициент трения

Коэффициент трения (также известный как коэффициент трения или коэффициент трения) представляет собой скалярную величину, используемую для расчета силы трения между двумя телами. Коэффициент трения зависит от используемых материалов — например, лед на металле имеет очень низкий коэффициент трения (они очень легко трутся друг о друга), в то время как резина на асфальте имеет очень высокий коэффициент трения (они не легко трутся друг о друга). ). Интересно отметить, что, вопреки распространенному мнению, сила трения не зависит от размера площади контакта между двумя объектами. Это означает, что трение не зависит от размера объектов. Сила трения всегда действует в направлении, противоположном движению. Например, стул, скользящий по полу вправо, испытывает силу трения в левом направлении.


Типы трения

Статическое трение

Статическое трение возникает, когда два объекта не движутся относительно друг друга (например, стол на земле). Коэффициент статического трения обычно обозначается как μ. В начальной силе, заставляющей объект двигаться, часто преобладает статическое трение.

Кинетическое трение

Кинетическое трение возникает, когда два объекта движутся относительно друг друга и трутся друг о друга (как сани о землю). Коэффициент кинетического трения обычно обозначается как μ и обычно меньше коэффициента статического трения.

Трение скольжения

Это когда два объекта трутся друг о друга. Положите книгу на стол и передвигайте ее — это пример трения скольжения.


4.4 Крутящий момент, передаваемый муфтой

Максимальный крутящий момент, передаваемый муфтой, определяется фрикционным материалом накладки, средним радиусом накладки (с обеих сторон) и давлением пружины пластина. Масло или смазка на накладке, которые уменьшат трение, или слабые или сломанные пружины в нажимном диске могут привести к проскальзыванию сцепления под давлением.

Теперь ясно видно, что средний радиус футеровки А на 10 % больше, чем футеровки В. Это означает, что футеровка А может передавать больший крутящий момент на целых 10 %. Пример ширины футеровки призван развеять мнение, что увеличение площади позволяет передавать больший крутящий момент. Подходящей шириной футеровки является ширина, достаточно узкая, чтобы обеспечить наибольший средний радиус, но не настолько узкая, чтобы допустить быстрый износ или выцветание.


Факторы, влияющие на передачу крутящего момента

Для передачи крутящего момента муфтой без проскальзывания необходимо учитывать четыре фактора.

  • Количество поверхностей (S).
  • Суммарное давление пружины (P).
  • Коэффициент трения (мк).
  • Средний радиус.


Крутящий момент = Цилиндр

(с) Две поверхности. Давление пружины (Н). Коэффициент трения (μ), 100 мм = 1 м (радиус)


Неисправность

Причина

Муфта пробуксовки

Изношенная подкладка
Недостаточный свободный ход педали сцепления.
Масло или смазка на фрикционных накладках
Слабые нажимные пружины.
Чрезмерные задиры на поверхности маховика из-за износа накладки.

Фрикцион сцепления Ведущий диск не освобождается при нажатии на педаль

Деформированный ведущий диск
Неправильная регулировка педали приводит к недостаточному перемещению выжимного подшипника.
Масло или смазка на фрикционной накладке.
Ведущий диск (диск сцепления) заклинило на шлицах.
Сломаны рычаги разблокировки.

Вибрация сцепления

Изношенная накладка или торчащие заклепки.
Масло на накладках.
Деформированный приводной диск.
Ослаблены крепления двигателя или коробки передач или ослаблены рулевые тяги.

0002  

Если вы являетесь автором приведенного выше текста и не согласны делиться своими знаниями в целях обучения, исследований, стипендий (для добросовестного использования, как указано в законе об авторских правах США), отправьте нам электронное письмо, и мы быстро удалить текст. Добросовестное использование — это ограничение и исключение исключительного права, предоставленного авторским правом автору творческого произведения. В законе США об авторском праве добросовестное использование — это доктрина, которая разрешает ограниченное использование материалов, защищенных авторским правом, без получения разрешения от правообладателей. Примеры добросовестного использования включают комментарии, поисковые системы, критику, новостные репортажи, исследования, обучение, библиотечное архивирование и стипендию. Он предусматривает законное нелицензионное цитирование или включение материалов, защищенных авторским правом, в работу другого автора в соответствии с четырехфакторным тестом баланса. (источник: http://en.wikipedia.org/wiki/Fair_use)

Информация о медицине и здоровье, содержащаяся на сайте, носит общий характер и цель, которая является чисто информативной и по этой причине ни в коем случае не может заменить совет врача или квалифицированного лица на законных основаниях профессия.

Тексты являются собственностью их соответствующих авторов, и мы благодарим их за предоставленную нам возможность бесплатно делиться своими текстами со студентами, преподавателями и пользователями Интернета, которые будут использоваться только в иллюстративных образовательных и научных целях.

Описание центробежной муфты – Руководство инженера по центробежной муфте

 

Что такое центробежная муфта?

Центробежная муфта представляет собой механическое устройство, которое используется на приводном вращающемся оборудовании. Муфта, чаще всего используемая с двигателем внутреннего сгорания, может использоваться для автоматической передачи крутящего момента от привода к ведомому оборудованию, обеспечивая «мягкий пуск» без включения нагрузки. Используя этот тип муфты между приводом и ведомым оборудованием, можно контролировать скорость, с которой зацепляется механический ведомый вал. Когда частота вращения двигателя увеличивается до установленной скорости включения центробежной муфты или превышает ее, включается механический привод. Это позволяет оператору запускать двигатель на заданной скорости холостого хода, не приводя в движение оборудование, что позволяет двигателю достичь оптимального крутящего момента до того, как он испытает нагрузку.

 

 

Каковы преимущества и недостатки использования центробежного сцепления?

Использование центробежной муфты на оборудовании с приводом от двигателя позволяет запускать двигатель на холостом ходу. Когда двигатель работает на холостом ходу, привод остается отключенным. Только при увеличении оборотов двигателя до установленной скорости включения сцепления или выше привод будет полностью подключен. Это приводит к плавному включению и предотвращает остановку двигателя. Это также помогает защитить двигатель, гарантируя, что высокие уровни крутящего момента не будут передаваться обратно через маховик двигателя. В таких обстоятельствах, например, когда вращающееся оборудование по какой-либо причине заклинило, экстремальные уровни крутящего момента, передаваемого обратно через двигатель, могут привести к значительному, а в некоторых случаях и непоправимому повреждению двигателя. Это может быть очень затратным, а иногда и экономически нецелесообразным ремонт оборудования. Этого можно избежать, используя центробежную муфту, так как компоненты муфты будут воспринимать перегрузку. Изнашиваемые детали сцепления легко и недорого заменяются. Помимо обеспечения ситуации холостого хода, заданная скорость включения сцепления также позволяет оператору контролировать, в какой момент включается вращающееся оборудование. Это позволяет двигателю машины работать, но не обязательно работать.

В связи с тем, что по своей природе центробежная муфта представляет собой чисто механическое автоматическое зацепление на заранее определенной скорости, для каждого применения может потребоваться определенная конфигурация. Это означает, что эту предварительно установленную скорость включения нельзя изменить без изменения внутренних компонентов.

 

Как работает центробежная муфта?

Центробежная муфта работает, как следует из названия, за счет центробежной силы. Ключевыми компонентами центробежного сцепления являются ступица, противовесы (колодки сцепления), пружины, накладки и корпус (показаны на схеме ниже). Центробежная сила, создаваемая оборотами двигателя, передается через два и более грузика. Муфта может приводиться в действие несколькими способами в зависимости от конструкции машины. Одним из наиболее распространенных методов является установка сцепления на параллельный или конический коленчатый вал двигателя. Когда коленчатый вал вращается, вал сцепления вращается с той же скоростью, что и двигатель. Вращение ступицы выталкивает башмаки или грузики наружу до тех пор, пока они не соприкоснутся с барабаном сцепления, фрикционный материал передает крутящий момент от грузиков на барабан. После этого диск подключается. Пружины, грузики и фрикционный материал определяют скорость, при которой включается сцепление. В зависимости от конструкции машины выходом из сцепления может быть один из множества приводов, включая, помимо прочего, вал, шкив, звездочку или фланец.

 

 

Применение центробежной муфты

Центробежная муфта может быть полезна для целого ряда машинного оборудования с высокой инерцией при запуске. Они обычно встречаются на мобильном оборудовании с вращающимися частями, которые приводятся в движение небольшими дизельными или бензиновыми двигателями. Некоторые из этих примеров включают:

  • Виброплиты и катки
  • Трамбовки
  • Приводы компрессоров/вакуумов/вентиляторов
  • Мастерки и шпатели по бетону
  • Компактные дорожные/уборочные машины
  • Транспортные холодильные установки
  • Мобильные водяные насосы
  • Оборудование для ухода за землей – роторные и цеповые косилки, газонокосилки и скарификаторы
  • Карты
  • Дробилки/измельчители пней/фрезы

 


 

Свяжитесь с нами

Наша команда специалистов разрабатывает и поставляет полный спектр центробежных муфт Amsbeck GmbH. Посетите нашу страницу «Центробежные муфты» для получения дополнительной информации или свяжитесь с нами, используя контактную информацию ниже.

Тел.: +44(0)1484 606040

Электронная почта: [email protected]

Недавние должности

  • Национальная неделя ученичества 2020

  • Планетарные передачи: Принципы работы

  • Обучение по обслуживанию передач для технического обслуживания в мировой компании по производству нефтяных услуг

Что такое сцепление и как работает сцепление?

Что такое сцепление?

Сцепление — механическое устройство, которое включает и выключает передачу мощности, в частности, с ведущего вала (ведущего вала) на ведомый вал. Сцепление действует как механическое соединение между двигателем и коробкой передач. И на короткое время отключает или отделяет двигатель от трансмиссии и, следовательно, от ведущих колес при нажатии педали, позволяя водителю плавно переключать передачи.

В простейшем случае муфты соединяют и разъединяют два вращающихся вала (ведущие валы или линейные валы). Эти устройства обычно имеют один вал, прикрепленный к двигателю, а другой — к силовому агрегату (приводному элементу). В то время как другой вал (ведомый элемент) обеспечивает выходную мощность для работы, и обычно движения вращаются, линейные муфты также возможны.

Например, в дрели с регулируемым крутящим моментом один вал приводится в движение двигателем, а другой — сверлильным патроном. Муфта соединяет два вала таким образом, что они запираются вместе и вращаются с одинаковой скоростью (зацеплены), запираются вместе, но вращаются с разными скоростями (проскальзывание) или разблокируются и вращаются с разными скоростями (отцепляются).

Что делает автомобильное сцепление?

Все знают, что у автомобиля есть двигатель, но не все знакомы со сцеплением или с тем, как оно работает. Этот механизм включает и отключает передачу мощности от ведущего вала к ведомому валу.

Он соединяет вращающиеся валы, и их может быть два или более под капотом. Если вы управляете механической коробкой передач, сцепление связано как с валом, идущим от двигателя, так и с валами, вращающими колеса. В то время как двигатель будет вращаться постоянно, вы не хотите, чтобы колеса постоянно вращались.

Один из вращающихся валов будет соединен с двигателем или силовым агрегатом, он будет ведущим элементом, а другой вращающийся вал или ведомый элемент обеспечивает работу. Например, у дрели есть вал, который приводится в движение двигателем, и вал, который приводится в движение сверлильным патроном.

Муфта соединяет валы таким образом, что они могут быть зацеплены (вращаться с одинаковой скоростью), пробуксовывать (вращаться с разными скоростями) или расцепляться (вращаться с разными скоростями). Как правило, вы обнаружите, что эти движения вращательные; хотя возможны линейные муфты.

Как работает автомобильное сцепление?

Он передает мощность двигателя на коробку передач и позволяет прерывать передачу при выборе передачи для трогания с места или при переключении передач во время движения автомобиля.

В большинстве автомобилей используется фрикционная муфта, приводимая в действие либо жидкостью (гидравлически), либо, чаще, тросом.

Когда автомобиль движется под нагрузкой, сцепление включено. Нажимной диск, прикрепленный болтами к маховику, оказывает постоянную силу с помощью диафрагменной пружины на ведомый диск.

В более ранних автомобилях вместо диафрагменной пружины в задней части прижимной пластины был ряд спиральных пружин.

Ведомая (или фрикционная) пластина движется по шлицевому входному валу, через который мощность передается на коробку передач. Пластина имеет фрикционные накладки, аналогичные тормозным накладкам, с обеих сторон. Это позволяет плавно включать передачу при включенном сцеплении.

Когда сцепление выключено (педаль нажата), рычаг прижимает выжимной подшипник к центру диафрагменной пружины, что снижает прижимное давление.

Внешняя часть нажимного диска, имеющая большую поверхность трения, больше не прижимает ведомый диск к маховику, поэтому передача мощности прерывается и можно переключать передачи.

При отпускании педали сцепления упорный подшипник выдвигается, и нагрузка диафрагменной пружины снова прижимает ведомый диск к маховику, возобновляя передачу мощности.

Некоторые автомобили оснащены сцеплением с гидравлическим приводом. Давление на педаль сцепления внутри автомобиля приводит в действие поршень в главном цилиндре, который передает давление через трубку, заполненную жидкостью, на рабочий цилиндр, установленный на картере сцепления.

Поршень рабочего цилиндра соединен с рычагом выключения сцепления.

Детали сцепления

Современное сцепление состоит из четырех основных компонентов: кожуха (с диафрагменной пружиной), нажимного диска, ведомого диска и выжимного подшипника.

Крышка крепится болтами к маховику, а нажимной диск оказывает давление на ведомый диск через диафрагменную пружину или винтовые пружины на более ранних автомобилях.

Ведомая пластина движется по шлицевому валу между прижимной пластиной и маховиком.

С каждой стороны он покрыт фрикционным материалом, который захватывает нажимной диск и маховик при полном включении и может проскальзывать на контролируемую величину при частичном нажатии на педаль сцепления, обеспечивая плавное включение привода.

Типы сцепления

Ниже приведены различные типы сцепления:

  • Фрикционное сцепление — однодисковое сцепление | Многодисковое сцепление — мокрое и сухое сцепление | Конусная муфта
  • Центробежная муфта
  • Полуцентрифугальная сцепление
  • Гидравлическая муфта
  • Коническое пружинное сцепление или сцепление диафрагмы
  • положительное сцепление или сцепление собаки и сплайна
  • вакуумная сцепление
  • Электромагнетическое сцепление

1)

Frict используют базовую фрикционную муфту, которая в основном имеет некоторые нормальные компоненты, о которых люди могли слышать раньше. Инженеры могут использовать фрикционную муфту для включения и выключения трансмиссии и маховика.

Приводится в действие с помощью механического троса или гидравлического троса, состоящего из диска сцепления, прижимного диска и выжимного подшипника.

Типы фрикционной муфты

Она делится на две части. К ним относятся:

  1. Однодисковое сцепление и
  2. Многодисковое сцепление
Однодисковое сцепление:

Однодисковое сцепление в основном используется в легковых автомобилях для передачи крутящего момента от двигателя к первичному валу. Судя по названию этого сцепления, у него всего одна пластина сцепления.

Многодисковое сцепление:

Этот тип сцепления имеет несколько дисков сцепления, которые используются для передачи мощности от вала двигателя к валу коробки передач того же автомобиля.

Также делится на две части; это мокрое сцепление и сухое сцепление. Вот крутое видео о мокром и сухом сцеплении [Внешняя ссылка]!

Сцепление, работающее в масляной ванне, называется мокрым сцеплением. С другой стороны, сухое сцепление работает без масла.

Принцип работы фрикционной муфты:

В автомобиле расцепление между двигателем и коробкой передач происходит путем приложения силы к муфте, таким образом, пружины сжимаются педалью, а нажимной диск сдвигается назад.

После этой ситуации диск сцепления освободился между маховиком и нажимным диском. Теперь сцепление может переключать передачи.

Принцип сцепления помогает вращать маховик до тех пор, пока вал двигателя не прекратит вращение. Сцепление отключает коробку передач и двигатель, так как оно было нажато водителем.

Кроме того, когда диск сцепления отпускается водителем, нажимной диск снова возвращается в исходное положение, и сцепление включается.

Однодисковое и многодисковое сцепление работают по одному и тому же принципу, хотя разница заключается в том, что однодисковое сцепление используется в легковых автомобилях, а многодисковое сцепление используется в большегрузных автомобилях.

2) Конусная муфта

Поверхность трения в этом типе муфт расположена в виде конуса, поэтому она называется конусной муфтой.

Две поверхности передают крутящий момент за счет трения. Вал двигателя состоит из охватываемого и охватывающего конуса. Он подразделяется на две секции: внутреннее и внешнее конусное сцепление.

  1. Конусы: охватывающий конус (зеленый), охватываемый конус (синий)
  2. Вал: охватываемый конус скользит по шлицам
  3. Фрикционный материал: обычно на охватывающем конусе, здесь на охватываемом конусе
  4. Пружина: приводит охватываемый конус назад после использования управления сцеплением
  5. Управление сцеплением: разделение обоих конусов нажатием
  6. Направление вращения: возможны оба направления оси
Преимущества конусной муфты

Вот несколько преимуществ конусной муфты:

  • По сравнению друг с другом конусная муфта более эффективна, чем одинарная пластинчатое сцепление.
  • В случае конической муфты на поверхность трения действует потенциал нормальной силы.
Недостатки конусной муфты

Хотя есть и недостатки конусной муфты, и вот они:

  • Конусная муфта часто неэффективна для отключения муфты.
  • Такая ситуация имеет место, когда угол больше 20°.
  • Небольшой износ может возникнуть из-за большого осевого смещения.

3)

Центробежная муфта

Для включения муфт центробежная муфта использует концепцию центробежной силы. Он работает автоматически в зависимости от скорости двигателя. Таким образом, в транспортном средстве для движения сцепления не требуется никакого лепестка сцепления.

Водитель может остановиться, а также запустить двигатель, не понижая и не повышая передачу.

Рекламные объявления

Принцип работы центробежной муфты
  • Эта муфта включает груз, который поворачивается в определенном месте. В соответствии с частотой вращения двигателя центробежная сила перемещает вес вверх и прикладывает усилие к коленчатому валу. За счет этого пластина прижимается.
  • После этого диск нажимает на пружину, которая в основном используется для нажатия на диск сцепления.
  • Теперь сцепление включено.
  • Сцепление остается выключенным до более низких оборотов, около 500 об/мин. Наконец, движение грузов контролируется кнопкой Stop (H).
Преимущества центробежного сцепления

Преимущества центробежного сцепления:

  • Автоматическое.
  • Низкая стоимость, а также низкая стоимость обслуживания.
  • Меньший износ.
  • Больше контроля над скоростью.
Недостатки центробежной муфты

Вот некоторые недостатки центробежной муфты:

  • Иногда двигатели страдают проскальзыванием на низких оборотах.
  • Нельзя использовать в высокоскоростном двигателе.
  • Пиковая скорость зависит от размера сцепления.

4)

Полуцентробежная муфта

Для удержания во включенном положении полуцентробежная муфта использует усилие пружины наряду с центробежной силой. Полуцентробежное сцепление состоит из диска сцепления, фрикционной накладки, рычагов, нажимного диска, маховика и пружин сцепления.

Принцип работы полуцентробежного сцепления
  • Пружина сцепления и рычаги одинаково закреплены на нажимном диске. При нормальной частоте вращения двигателя муфта предназначена для передачи крутящего момента пружине.
  • При нормальной скорости и низкой передаче мощности давление на нажимной диск отсутствует. Следовательно, сцепление остается включенным.
  • При высокой скорости и высокой мощности на нажимной диск оказывается давление, и сцепление включается.
  • Менее жесткие пружины сцепления помогают избавиться от напряжения при работе сцепления.
  • При снижении скорости автомобиля или резком падении скорости рычаги не оказывают давления на прижимную пластину.
Преимущества полуцентробежного сцепления
  • Менее жесткие пружины сцепления на малых оборотах.
  • Нет пятен для операций сцепления.
Недостатки полуцентробежного сцепления
  • При нормальной частоте вращения двигателя муфта предназначена для передачи крутящего момента пружине.
  • Способствует передаче крутящего момента на высокоскоростном двигателе за счет центробежной силы.

5)

Мембранная муфта

При включении муфт муфта этого типа создает давление на нажимной диск. Эта муфта выполнена из диафрагмы на конической пружине. Коронная или пальчиковая пружина прикреплена к прижимной пластине.

Принцип работы мембранной муфты
  • Для диафрагменной муфты мощность двигателя передается на маховик от коленчатого вала.
  • Маховик состоит из фрикционных накладок, а сцепление соединено с маховиком.
  • Поскольку на нажимной диск сцепления подается давление, за счет чего диск сцепления находится за нажимным диском.
  • Мембранная муфта конической формы. Наружный подшипник переходит на маховик после нажатия на педаль сцепления.
  • Внешний подшипник давит на мембранную пружину. Так что прижимная пластина толкается назад под действием диафрагменной пружины.
  • Это давление отключило сцепление, сняв давление на пластину.
  • Мембранная пружина и нажимной диск вернулись в нормальное состояние после сброса давления педалей сцепления.
Преимущества мембранной муфты
  • Вот некоторые преимущества мембранной муфты:
  • В мембранной муфте нет необходимости отпускать рычаги, так как пружины действуют как рычаги.
  • Спиральная пружина увеличивает давление больше, чем тяжелые лопасти. Чтобы не было нужды в тяжелых веслах.
Недостатки мембранной муфты
  • Так как муфта представляет собой конус, пружины становятся более жесткими, и для их расцепления требуется большее усилие.
  • При более высокой скорости спиральная пружина сталкивается с тенденцией деформации в поперечном направлении.

6)

Зубчатая и шлицевая муфта

Зубчатая и шлицевая муфта состоит из двух частей. Одно сцепление Dog, а другое сцепление Spline.

Сплайн также называют скользящей втулкой. Эта муфта используется для соединения вала с шестерней или для блокировки двух валов.

Принцип работы кулачковой и шлицевой муфты
  • Кулачковая муфта состоит из внешних зубьев, а шлицевая муфта состоит из внутренних зубьев.
  • Две муфты предназначены для вращения друг с другом с одинаковой скоростью, но они никогда не проскальзывают друг от друга.
  • Для зацепления двух валов они должны быть соединены. Скользящая втулка отходит от шлицевого вала назад и не соприкасается друг с другом, после чего сцепление выключается.
Преимущества кулачковой и шлицевой муфты
  • Муфты не соскальзывают друг с друга.
  • Собачья и шлицевая муфта создавала огромный крутящий момент.
  • Трение отсутствует, так как они сцепляются друг с другом при вращении.
  • Недостатки кулачковой и шлицевой муфты:
  • На более высокой скорости трудно включать и выключать муфты.
  • Для расцепления и зацепления требуется некоторое относительное движение.

7)

Электромагнитная муфта

Электромагнитная муфта изготовлена ​​из материалов, применяемых в электротехнике.

Это следующие:

  • Ротор: Ротор – это деталь, которая соединяется непосредственно с валом двигателя и помогает непрерывно вращать вал двигателя и ведущий вал.
  • Обмотка: Обмотка крепится за ротором. Он не вращается. Он подключен к источнику постоянного тока высокого напряжения, который с помощью обмотки преобразуется в электромагнит.
  • Якорь: Якорь крепится к передней части ротора. Он крепится к ступице болтами или заклепками.
  • Ступица: Ступица крепится за арматурой. Он крепится к ведомому валу болтами и вращается вместе с валом.
  • Фрикционная пластина: Основание на передаче мощности вставки фрикционной пластины между ротором и якорем.
  • Блок питания: Блок питания состоит из аккумулятора, выключателя сцепления, провода и т. д. Объявления
Принцип работы электромагнитной муфты
  • Высоковольтное питание постоянного тока подается на обмотку от динамо-машины или аккумулятора.
  • Обмотка создает электромагнитное поле, которое притягивает нажимной диск и включает сцепление.
  • Для отключения питание должно быть отключено.
  • Для перезапуска сцепления выполнен рычаг переключения передач, поэтому сцепление выключается переключением передач руками водителя.
  • Сцепление не включено, когда мощность динамо-машины низкая на низкой скорости.
  • На нажимном диске есть три пружины, которые также включают сцепление на низкой скорости.
Преимущества электромагнитной муфты
  • Процесс эксплуатации прост.
  • Дистанционное направление используется для управления сцеплением, поскольку для него не требуется рычажный механизм.
  • Недостатки электромагнитной муфты:
  • Высокая стоимость.
  • Поскольку никакие электрические компоненты не поддерживают высокую температуру, должно быть ограничение на рабочую температуру.

8)

Вакуумная муфта

Вакуумная муфта работает за счет вакуума. Итак, его название — Вакуумная муфта.

Состоит из таких частей. Those are:

  1. Switch
  2. Non-return valve
  3. Solenoid
  4. Piston
  5. Vacuum Reservoir
  6. Vacuum Cylinder
  7. Battery
  8. Inlet and Outlet
  9. vacuum clutch diagram
Working Principle of vacuum clutch
  • В коллекторе двигателя (впускном) имеется вакуум, который приводит в действие вакуумную муфту.
  • Коллектор двигателя соединяется через невозвратный клапан с вакуумным ресивером.
  • Резервуар подсоединяется через электромагнитный клапан с вакуумным цилиндром.
  • В рычаге переключения передач есть переключатель.
  • Аккумулятор приводит в действие соленоид.
  • Рычаг переключает передачу, когда он удерживается водителем и выполняется операция переключателя.
  • Повышение давления во впускном коллекторе при открытии дроссельной заслонки. Чтобы обратный клапан был закрыт, коллектор изолирует резервуар. В резервуаре все время присутствует вакуум.
Преимущества вакуумной муфты
  • Значительно дешевле других муфт.
  • Обеспечивает минимальный ход привода.
  • Недостатки вакуумной муфты:
  • Состоит из нескольких компонентов.
  • Иногда инженеры обнаруживают медлительность в машине.

9)

Гидравлическое сцепление

Принцип работы вакуумного и гидравлического сцепления практически одинаков.

Хотя существенная разница между ними заключается в том, что гидравлическая муфта работает за счет давления масла, с другой стороны, вакуумная муфта работает за счет вакуума.

Принцип действия гидромуфты

Масло подается в аккумулятор из резервуара с помощью насоса инженером. Соединение между аккумулятором и цилиндром осуществляется с помощью регулирующего клапана.

Двигатель автомобиля приводит в действие насос. Переключатель управляет клапаном. Кроме того, инженеры используют рычажный механизм для соединения поршня со сцеплением.

Водитель транспортного средства нажимает рычаг переключения передач транспортного средства и открывает переключатель клапана, чтобы обеспечить подачу масла. Под давлением масла поршень автомобиля начинает двигаться вперед и назад, что приводит к включению и выключению сцепления.

Преимущества гидравлического сцепления
  • Нажимать намного легче.
  • Предоставление эквивалентного количества жидкости.
  • Недостатки гидравлического сцепления:
  • Иногда из-за использования жидкостей силиконового типа может иметь место утечка.
  • Может повредить уплотнения.

10)

Муфта свободного хода

Ее часто называют по-разному, например, обгонная, односторонняя и пружинная муфта. Мощность передачи, создаваемая этими типами сцепления, в основном происходит в одном направлении.

Муфта свободного хода монтируется инженерами за коробкой передач двигателя.

Принцип работы муфты свободного хода

Ступица вышеупомянутой муфты вращается по часовой стрелке, после чего ролик поднимается вверх по кулачкам.

Это движение происходит за счет заклинивания. После этой ситуации за ступицей следует внешнее кольцо.

Гонщик вращается в том же направлении и с той же скоростью, что и ступица. Ступица соединена с главным валом, а наружная обойма соединена с выходным валом инженером.

Преимущества муфты свободного хода
  • Муфта свободного хода обеспечивает лучшую экономию топлива.
  • Меньший износ по сравнению с ручным сцеплением.
Недостатки муфты свободного хода

Если инженеры пытаются произвести торможение двигателя, то муфта свободного хода подвергается большему износу.

Материал сцепления

Существует так много материалов, которые использовались для изготовления дисков сцепления.

В прошлом для изготовления дисков сцепления использовался асбест. В настоящее время производители используют составную органическую смолу с медным покрытием проволоки, а также используют керамический материал.

При перевозке тяжелых грузов или гонках обычно использовались керамические материалы.

Сейчас в современном мире асбестовые отнесены к разряду ненадежных и вообще эти муфты не встречаются с современными усовершенствованными муфтами.

Полуметаллические материалы: Этот тип материала содержит от 30% до 65% стали, железа и меди. Эти муфты обладают высокой термостойкостью, их трудно сломать, и они достаточно прочны. Пластины надежны, но не очень хороши для высокоскоростной работы.

Органические материалы: Это наиболее распространенный тип материалов, который мы использовали чаще всего. Сцепления из этих материалов подходят для всех видов использования в различных транспортных средствах, таких как размер. Этот материал содержит большое количество меди, потому что он может эффективно передавать тепло.

Керамические материалы: Эти типы муфт содержат одновременно органические и неорганические материалы, включая стекло, резину, кевлар и углеродные материалы. В этом сцеплении коэффициент трения относительно высок и составляет от 0,33 до 0,4. Этот тип сцепления используется в самых интенсивных условиях, например, в грузовиках и гоночных автомобилях.

Часто задаваемые вопросы.

Что такое сцепление?

Сцепление — механическое устройство, которое включает и выключает передачу мощности, в частности, с ведущего вала (ведущего вала) на ведомый вал. Сцепление действует как механическое соединение между двигателем и коробкой передач. И на короткое время отключает или отделяет двигатель от трансмиссии и, следовательно, от ведущих колес при нажатии педали, позволяя водителю плавно переключать передачи.

Что такое сцепление в автомобиле?

Проще говоря, сцепление — это механическое устройство, передающее крутящий момент от двигателя к колесам любого транспортного средства с механической коробкой передач. Сцепление — это часть автомобиля, которая соединяет два или более вращающихся вала.

Какие существуют типы муфт?

Ниже приведены различные типы сцеплений:

  • Фрикционная муфта – однодисковая, многодисковая, мокрая и сухая, конусная
  • Центробежная муфта
  • Полуцентробежная муфта
  • Гидравлическая муфта
  • Муфта с конической пружиной или мембранная муфта
  • Муфта с принудительным приводом или муфта с собачкой и шлицом
  • Вакуумная муфта
  • Электромагнитная муфта

Что происходит, когда сцепление выходит из строя во время движения?

К счастью, выход из строя сцепления не приводит к катастрофическим последствиям, если сразу прекратить движение. Сломанное сцепление разрывает связь между двигателем и трансмиссией, в результате чего ваш автомобиль не может двигаться правильно. С механической коробкой передач вы все еще можете управлять своим автомобилем.

Важно ли сцепление в автомобиле?

Без правильно работающего сцепления мощность не передается должным образом, и, следовательно, автомобиль не будет переключаться. Двигатель в автомобиле постоянно вращается, поэтому сцепление действует как способ либо включить это вращение, либо отключить его для движения или остановки.

Можно ли водить машину без сцепления?

Как только включится переключение передач, слегка прибавьте газ для непрерывного и легкого вождения. Вождение автомобиля с неисправным сцеплением или без него может быть трудным и вредным для вашего автомобиля. Важно знать, как водить машину без сцепления. Убедитесь, что местный механик починил его, чтобы избежать проблем в будущем.

Что такое сцепление и зачем оно вам?

Поскольку ваш двигатель постоянно вращается, должен быть способ расцепления колес, чтобы они могли перестать двигаться. Вот где сцепление вступает в игру. Он может отключить колеса, не убивая двигатель.

У автомобилей с автоматическим сцеплением?

Автомобили как с механической, так и с автоматической коробкой передач имеют муфты, которые включают трансмиссию для направления мощности двигателя и движения колес автомобиля или отключают ее, чтобы остановить колеса, даже когда двигатель все еще работает. Вот несколько предупреждающих знаков, которые может дать вам сцепление.

Какие признаки неисправности сцепления?

8 Признаки неисправности сцепления

  • Слабое ускорение.
  • Шлифовальные шестерни.
  • Свободная педаль сцепления.
  • Залипающая педаль сцепления
  • Другие шумы.
  • Не переключается на передачу.
  • Не останется в снаряжении.
  • Запах гари.

Как понять, что сцепление вышло из строя?

Признаки неисправности сцепления:

  • Сцепление кажется губчатым, заедает или вибрирует при нажатии на него.
  • Вы слышите скрип или ворчание при нажатии на педаль.
  • Вы можете увеличить обороты двигателя, но ускорение плохое.
  • У вас проблемы с переключением передач.

Можно ли отремонтировать сцепление?

Если вы уверены, что ваше сцепление изношено и не подлежит ремонту, вы можете заменить его самостоятельно в домашних условиях. Это возможно, но это относительно длительная и сложная процедура. Есть несколько неудобных шагов, где есть широкий простор для того, чтобы что-то пойти не так.

Вы тормозите, потом сцепление?

Медленная езда, сцепление, затем тормоз при остановке. Двигайтесь быстрее, тормозите, затем выжимайте сцепление.

Что приводит к отказу сцепления?

Внезапный и постепенный отказ: Внезапный отказ чаще всего вызван обрывом или ослаблением троса сцепления, возможностью соединения или неисправностью главного/ведомого гидравлического цилиндра. Также могут быть утечки в гидравлической линии или даже диск может быть загрязнен грязью или мусором.

Сколько стоит замена сцепления?

Стоимость ремонта сцепления может составлять от 500 до 2500 долларов. Это действительно зависит от автомобиля. Замена сцепления в спортивных автомобилях, экзотических автомобилях и европейских автомобилях обходится дороже, чем в экономичных японских автомобилях. Полноприводные автомобили стоят дороже, чем двухколесные.

Как завести машину с неисправным сцеплением?

Новые автомобили имеют переключатель педали сцепления, который нужно нажать, чтобы автомобиль завелся. Даже если сцепление не выключается, нажмите на педаль, чтобы активировать переключатель, который позволяет стартеру запускать двигатель при повороте ключа. Держись, и ты ушел.

Как определить, сцепление это или коробка передач?

Выключите двигатель и посмотрите, сможете ли вы выбрать передачу. Если вы можете, то обычно проблема со сцеплением; если вы не можете, то проблема будет заключаться в коробке передач или рычажном механизме.

Каков срок службы автоматического сцепления?

Большинство сцеплений рассчитаны примерно на 60 000 миль пробега, прежде чем их потребуется заменить. Некоторым может потребоваться замена через 30 000 км пробега, а некоторые могут продолжать работать более 100 000 миль, но это довольно редко.

Есть ли в современных автомобилях сцепление?

В современных автомобилях F1 есть сцепления, и их включение разрывает связь между двигателем и коробкой передач, что позволяет переключать передачи. Это также позволяет плавно трогаться с места и останавливаться, не останавливая двигатель и не повреждая коробку передач.

Как долго должно работать сцепление?

Водитель должен выжать сцепление для переключения передач. Средний срок службы сцепления составляет от 20 000 до 150 000 миль. К счастью, ваше сцепление, скорее всего, даст вам достаточно информации о том, что что-то идет не так.

Можно ли выжимать сцепление при торможении?

При торможении всегда следует выжимать сцепление. Что ж, остановка автомобиля, особенно с нагрузкой на трансмиссию, серьезно повлияет на коробку передач в сборе. Поэтому всегда рекомендуется выжимать сцепление при торможении, по крайней мере, в начале движения.

Нужно ли нажимать сцепление при повороте?

Нет, нажимать сцепление при поворотах нельзя. Выжатое сцепление освобождает передаточный механизм, что влияет на устойчивость кузова автомобиля. Когда вы поворачиваете, и ваше транспортное средство выходит из равновесия, оно может потерять сцепление с дорогой и, в зависимости от скорости, транспортное средство может опрокинуться.

Как замедлить машину?

Использование сцепления для замедления работает в паре с педалью акселератора. Отпустите газ и дайте машине немного замедлиться. Затем нажмите сцепление, переключитесь на пониженную передачу и снова отпустите сцепление. Если вы правильно рассчитаете время, вы почувствуете твердое, но плавное замедление.

Где вы используете сцепление?

Муфта частично предназначена для обеспечения такого контроля; в частности, муфта обеспечивает передачу крутящего момента между валами, вращающимися с разной скоростью. В крайнем случае управление сцеплением используется при динамичном вождении, например, при трогании с мертвой точки, когда двигатель создает максимальный крутящий момент на высоких оборотах.

Как узнать, когда переключать передачи?

Как правило, повышать передачу следует, когда тахометр показывает отметку «3» или 3000 об/мин; переключаться вниз, когда тахометр находится около «1» или 1000 об/мин. После некоторого опыта вождения рычага переключения передач вы сможете понять, когда переключаться, по тому, как звучит и «ощущается» ваш двигатель. Подробнее об этом ниже.

Муфта | Определение, требование, функция, тип

Адитья 2 комментария центробежное сцепление, сцепление, конусное сцепление, мембранное сцепление, функция сцепления, функция сцепления в автомобиле, многодисковое сцепление, полуцентробежное сцепление, однодисковое сцепление, типы сцепления, типы сцепления pdf

Содержание

Определение и необходимость для сцепления

Сцепление – это устройство, используемое в системе трансмиссии транспортного средства для передачи вращательного движения одного вала при необходимости на второй вал, ось которого совпадает с осью первого. Таким образом, сцепление находится между двигателем и системой трансмиссии.
Когда сцепление включено, мощность передается от двигателя к ведущим колесам через систему трансмиссии, и автомобиль движется. При выключенном сцеплении мощность не передается на ведущее колесо и на остановку автомобиля, пока работает двигатель.
В автомобиле сцепление всегда находится во включенном положении. Сцепление выключается при переключении передач, при остановке автомобиля или при торможении и при работе двигателя на холостом ходу. Он должен отключаться нажатием педали сцепления.
При правильной работе сцепления предотвращается резкое рывковое движение автомобиля, что позволяет избежать нагрузки на остальные части силовой передачи.

Зачем нужна муфта

1) Она должна передавать максимальный крутящий момент двигателя.
2) Включать следует постепенно, чтобы избежать резких рывков.
3) Он должен рассеивать большое количество тепла, выделяемого при работе сцепления.
4) Должен быть динамически сбалансирован, для высокоскоростного двигателя.
5) Он должен иметь подходящий механизм для гашения вибрации и снижения шума при передаче мощности.

Принцип работы

Муфта представляет собой механизм, используемый для передачи вращательного движения от одного вала к другому, когда это необходимо, и оси обоих валов совпадают.

Работает по принципу трения. Он соединяет вал двигателя и вал коробки передач. На передачу мощности может влиять трение между двумя или более вращающимися концентрическими поверхностями, называемыми фрикционными дисками. Фрикционные пластины могут плотно прижиматься друг к другу за счет осевого усилия, создаваемого пружиной и прижимной пластиной. Когда сцепление сцеплено и сцепление имеет тенденцию вращаться как единое целое, скорость вращения концентрической поверхности или другого вала зависит от силы трения, которая пропорциональна осевой силе, приложенной пружиной.

Назначение

1) Для включения и выключения передачи на неподвижном автомобиле с работающим двигателем.
2) Передавайте мощность двигателя на опорное колесо плавно, без ударов в систему трансмиссии, приводя транспортное средство в движение.
3) Для включения передачи во время движения автомобиля без демпфирования шестерни.

Однодисковое сцепление

Конструкция и работа


Оно состоит из маховика, нажимного диска, диска сцепления с фрикционными накладками/поверхностями, называемыми фриктонными дисками, педали сцепления, упорного шарикоподшипника и пружин сцепления.
Он имеет только один диск сцепления, установленный на ступице, которая имеет внутренние шлицы и может свободно скользить по валам сцепления.
Маховик установлен на коленчатом валу и вращается вместе с ним.
Нажимной диск крепится к маховику через пружину сцепления с помощью болтов.
Пружины расположены по окружности, чтобы обеспечить осевое усилие на прижимной диск, чтобы удерживать сцепление во включенном положении.
Фрикционная пластина, удерживаемая между маховиком и нажимной пластиной, имеет фрикционную поверхность с обеих сторон для обеспечения двух кольцевых фрикционных поверхностей для передачи мощности.

При нажатии на педаль сцепления нажимной диск перемещается вправо против усилия пружины, что достигается с помощью подходящего рычажного механизма и упорного подшипника.
Из-за этого движения нажимного диска фрикционный диск освобождается и сцепление выключается, а когда педаль отпускается, нажимной диск перемещается влево из-за пружины сцепления, расположенной по окружности, и, таким образом, сцепление снова включается.

Преимущества
Переключение передач проще, чем у конусной муфты, потому что педаль движется меньше.
Надежнее, так как не страдает недостатком заедания конуса.

Недостаток
Пружины должны быть более жесткими, поэтому водителю требуется большее усилие для расцепления.

Применение
Это сцепление используется в четырехколесных транспортных средствах, таких как грузовики, джипы, Fiat-1100, Tata sierra и т. д.

Многодисковое сцепление и диск сцепления).


Из-за увеличения поверхности трения мощность передачи крутящего момента муфты увеличилась, а размер остался фиксированным. В качестве альтернативы размер муфты уменьшился для передачи той же мощности, что и однодисковая муфта.
Этот тип сцепления используется в большегрузных транспортных средствах и гоночных автомобилях, где передается высокий крутящий момент.
Это сцепление также используется в скутерах и мотоциклах, так как доступное место ограничено.
Состоит из маховика, нажимного диска, втулки с пружиной сцепления, упорного подшипника и педали сцепления, а также двух комплектов фрикционных дисков.
Один комплект фрикционных дисков скользит в продольной канавке маховика, а другой скользит по шлицам на ступице нажимного диска.
Фрикционные диски расположены поочередно в каждом комплекте.
Когда педаль сцепления нажата, осевое давление воздействует на нажимной диск, перемещая его вправо и сжимая пружину сцепления на конце втулки.
Таким образом, давление на фрикционную пластину ослабевает, и она выходит из зацепления.
Когда педаль сцепления отпущена, пружина сцепления оказывает давление на нажимной диск, перемещая нажимной диск влево и, таким образом, сцепление снова включается.

Преимущества многодискового сцепления
Уменьшение веса сцепления.
Имеет очень компактные размеры.
Увеличьте величину передаваемого крутящего момента.
Уменьшить момент инерции сцепления.

Недостатки многодискового сцепления
Быстро нагреваются.
Многодисковые муфты тяжелые.
Многодисковые муфты слишком дороги.

Применение многодискового сцепления
Многодисковые сцепления используются там, где требуется компактная конструкция, например, в скутерах, мотоциклах и гоночных автомобилях.
Многодисковое сцепление используется в некоторых тяжелых коммерческих автомобилях для передачи высокого крутящего момента.

Мембранная муфта

В этой муфте для создания необходимого давления для ее включения вместо винтовой пружины используется диафрагма или коническая пружина.
Этот тип сцепления довольно удобен, поскольку не требует рычагов растормаживания, а сама пружина действует как ряд рычагов.
Некоторые из используемых конических пружин даже не имеют характеристик постоянной жесткости.
Давление пружины постоянно меняется.
Увеличивается до тех пор, пока пружина не достигнет своего плоского положения, и уменьшается при прохождении этого положения.
В случае с этим сцеплением водителю не нужно прилагать такое сильное давление на педаль, чтобы удерживать сцепление вне зацепления, как в случае с цилиндрической пружиной.
В случае сцепления со спиральной пружиной при нажатии педали для выключения сцепления давление пружины еще больше увеличивается.
Пружина поворачивается на задних поворотных кольцах в положении включения, удерживая себя на крышке сцепления.
В этом положении нажимная пластина касается своего внешнего обода. Таким образом, пружина оказывает достаточное давление, создавая прочный контакт между нажимным диском и диском сцепления, а также с маховиком в этом естественном коническом положении.
Теперь для выключения сцепления педаль нажимается.
Подшипник перемещается к маховику рычажным механизмом из-за нажатия педали.
Поскольку пружина поворачивается на переднем шарнирном кольце, подшипник, контактирующий с внутренней частью конической пружины, перемещает эту часть вперед, в результате чего обод движется назад.
Диск сцепления выходит из контакта с обоими ведущими элементами, потому что давление на нажимной диск снимается.
Рычаги выключения будут перемещаться в сторону извлеченного подшипника и препятствовать полному включению сцепления из-за износа фрикционной накладки.
Для предотвращения проскальзывания сцепления предусмотрен свободный ход около 25 мм у педали сцепления или 1,5 мм у выжимного подшипника.

Мембранная муфта Преимущества
Легче по весу.
Компактный размер.
Мембранная пружина прикладывает большее усилие, поэтому она имеет более высокую способность передачи крутящего момента.
В нем меньше вращающихся частей, поэтому нет проблем с шумом при работе.
Имеет динамический крен даже на высокой скорости.

Мембранная муфта Недостатки
Для большегрузных автомобилей размер муфты увеличивается для увеличения поверхностей трения.

Мембранная муфта имеет следующие области применения
maruri suzuki swift
tata storm safari
Ford Eco Sport

Полуцентробежное сцепление

В этом типе сцепления используется легкая пружина сцепления достаточной прочности, оказывающая низкое давление на холостых оборотах.
Также увеличивается давление между фрикционными дисками с увеличением скорости вращения муфты пропорционально требуемому давлению с помощью центробежной массы, прикрепленной к шарнирному рычагу с равным интервалом.
Нажимной диск перемещается к маховику, а концы рычага выключения отодвигаются назад к выжимному подшипнику под действием центробежной силы, которая увеличивается с увеличением скорости вращения.
Давление пружины сцепления оказывает низкое давление на холостом ходу. Этот набор может использоваться для облегчения нажатия педали сцепления для переключения передач.
По мере увеличения скорости вращения вращающийся груз стремится сместить прижимной диск к маховику.
Концы рычагов расцепления также перемещаются назад к выжимному подшипнику вместе с этим движением прижимной пластины.

Преимущества
Управление сцеплением очень простое.
Используются менее жесткие пружины сцепления, поскольку они работают только при низких оборотах двигателя.

Недостатки
Пружины передают крутящий момент только при более низких оборотах двигателя.
Центробежные силы работают только при более высоких оборотах двигателя для передачи крутящего момента.

Применение
Полуцентробежное сцепление используется в автомобилях Vauxhall.

Центробежное сцепление

Центробежное сцепление широко используется в легких двухколесных транспортных средствах, таких как мопед или двухколесный транспорт без коробки передач.
Центробежная муфта использует центробежную силу, а не силу пружины для удержания ее во включенном положении, для управления муфтой не требуется педаль сцепления.
Сцепление включается автоматически в зависимости от частоты вращения двигателя.
Автомобиль можно остановить на передаче без остановки двигателя, а водитель может завести автомобиль, выбрав любую передачу.
Состоит из маховика, нажимной пластины, фрикционной пластины, пружины, крышки со стопором и коленчатого рычага.
При нажатии на педаль акселератора обороты двигателя увеличиваются. Груз A, прикрепленный к одному концу коленчатого рычага, летит вверх, оказывая давление на прижимную пластину C через пружину E.
Фрикционная пластина D прижимается к маховику F, включая сцепление.
Пружина G служит для удержания сцепления в выключенном состоянии на малой скорости. H — стопор для ограничения величины центробежной силы.

Преимущества
Он прост и требует меньше обслуживания.
дешевле
Так как он автоматический, то и не нужен необходимый механизм управления.
Скорость соединения можно регулировать, выбрав соответствующую пружину.
Помогает предотвратить остановку двигателя.

Недостатки
В нем потеря мощности из-за проскальзывания и трения.
Не способен передавать большую мощность и проскальзывает в условиях большой нагрузки.
Проблема с перегревом.
Их сцепление зависит от скорости приводного вала.

Применение центробежной муфты
Центробежная муфта может быть полезна для целого ряда машинного оборудования с высокой инерцией при запуске. Они обычно встречаются в мобильном оборудовании с вращающимися частями, которые приводятся в действие небольшими дизельными или бензиновыми двигателями. Центробежное сцепление в основном используется в газонокосилках, мопедах, картингах, мини-велосипедах, бензопилах и т. д. Некоторые из этих примеров включают

Виброплиты и катки
Трамбовки
Компрессорно-вакуумные/вентиляторные агрегаты
Бетоно-затирочные машины
Компактные уборочные машины
Транспортные холодильные установки
Мобильные водяные насосы
Садовая техника: роторные и цеповые косилки, газонокосилки и скарификаторы 900
Измельчители древесины/измельчители пней/фрезерные станки

Конусная муфта

Конусная муфта состоит из фрикционных поверхностей в форме конуса.
Вал двигателя состоит из охватывающего конуса и охватываемого конуса, установленного на валу сцепления, который является шлицевым, и охватываемый конус может свободно скользить по валу сцепления.
Охватываемый конус снабжен фрикционными поверхностями.
Когда охватываемый конус соприкасается с охватывающим конусом, муфта включается под действием силы пружины при нажатии на педаль сцепления, охватываемый конус скользит вправо против усилия пружины, сжимая пружину, и муфта отключается.
Конусная муфта практически устарела из-за некоторых недостатков.
Если угол конуса сделать меньше 20 градусов, охватываемый конус имеет тенденцию застревать в охватывающем конусе, и его становится трудно зацепить.

Преимущества
В этой муфте нормальная сила, действующая на контактные поверхности, больше осевой силы.
Может передавать высокий крутящий момент по сравнению с дисковыми муфтами того же размера.
Поскольку контактные поверхности имеют клиновидную форму, для включения сцепления требуется меньшее усилие.
Создает меньше шума, чем другие диски сцепления.

Недостатки
Если угол конуса большой, усилие, необходимое для выключения сцепления, будет больше.
Даже небольшой износ поверхности конуса нарушает правильную работу сцепления.
Если угол конуса становится меньше примерно 20°, охватываемый конус имеет тенденцию связываться или связываться с охватывающим конусом, и становится трудно расцепить муфту.

Конусная муфта Применение
Конусная муфта используется в различных механических коробках передач в качестве синхронизаторов.
Они используются в различных тяжелых машинах, так как могут передавать высокий крутящий момент.
Эти муфты обычно используются в устройствах с низкой окружной скоростью.
Конусные муфты обычно используются на моторных гоночных лодках. Конусные муфты
используются в гоночных и раллийных автомобилях.

Часто задаваемые вопросы

Какова функция сцепления?
Функцией и назначением сцепления является передача крутящего момента от вращающегося двигателя к коробке передач. Муфты требуют режима срабатывания, чтобы разорвать передачу крутящего момента.

Из каких частей состоит сцепление?
Сцепление состоит из четырех частей:
Диск сцепления.
Обложка.
Подшипник (выжимной)
Вилка выключения (вилка сцепления)

Где находится сцепление?
Сцепление находится посередине трансмиссии и двигателя. Так что крутящий момент может передаваться от двигателя к трансмиссии, когда это необходимо.

Как изготавливается сцепление?
Эти муфты обычно состоят из набора чередующихся фрикционных и стальных пластин. Фрикционные диски имеют выступы на внешнем диаметре, которые фиксируют их в корзине, вращаемой коленчатым валом. Стальные пластины имеют выступы на внутреннем диаметре, которые фиксируют их на входном валу коробки передач.

Вы также можете прочитать

  • Что такое Gears.
  • Что такое Gear Терминология.

Если вы хотите видео на эту тему, нажмите ссылку ниже

  • Как работает сцепление в автомобиле с анимацией

Типы сцепления и их части двигатель от колес во время движения с переключением передач.

Благодаря сцеплению мы можем добиться правильной передачи мощности двигателя от коробки передач, которая затем передается на колеса автомобиля.

В этом посте Frenkit мы объясняем различные типы клатчей, которые существуют на рынке, все они отличаются друг от друга, но служат одной и той же цели. Здесь мы расскажем вам о них все, вы не можете их пропустить!

 

Типы сцепления в автомобилях

Система сцепления в основном является частью системы сцепления автомобилей, оснащенных коробками передач. Как упоминалось в предыдущих постах, сцепление в автомобиле «обеспечивает правильную передачу мощности от двигателя к коробке передач, которая затем передается на колеса автомобиля».

В зависимости от характеристик каждой модели автомобиля мы можем различать разные виды сцепления. Например, мы можем различать систему сцепления по дискам, которая может быть гидравлической, однодисковой, двухдисковой или многодисковой; по типу охлаждения бывают сухие или мокрые сцепления; или тип управления, которые, среди прочего, делятся на фрикционные, электромагнитные или гидравлические муфты.

Далее мы подробно опишем различные типы приводов сцепления, которые мы упомянули выше и которые доступны на рынке. Каждый из них имеет свои особенности.

По дискам

  • Гидромуфта . Для правильной работы сцепления этого типа необходимо, чтобы гидравлическая жидкость приводилась в действие. По этой причине гидравлическое сцепление не имеет дисков, так как мощность, необходимая для привода автомобиля, вырабатывается турбиной, которая приводится в движение за счет вращения двигателя автомобиля. В частности, этот тип сцепления не используется из-за большого расхода топлива и больших затрат на поддержание автомобиля в рабочем состоянии. Тем не менее, это тип сцепления, который можно увидеть в промышленных транспортных средствах.
  • Однодисковое сцепление . Этот тип сцепления называется однодисковым, поскольку между маховиком двигателя и нажимным диском имеется только один диск.
  • Двухдисковое сцепление . В данном случае эта модель сцепления имеет два диска для правильной передачи мощности от двигателя к коробке передач. Двухдисковую систему сцепления можно найти в автомобилях, которые имеют тенденцию генерировать довольно большую мощность.
  • Многодисковое сцепление. Эта модель сцепления состоит из ряда фрикционных дисков, которые отвечают за управление передачей мощности в автомобиле. Многодисковое сцепление можно найти, например, в мотоциклах.

По типу охлаждения

  • Сухое сцепление . В зависимости от типа охлаждения системы сцепления мы можем обнаружить, что охлаждение является воздушным, в случае сухого сцепления.
  • Мокрое сцепление . Когда охлаждение осуществляется маслом, речь идет о мокром типе сцепления.

По типу управления

  • Фрикционная муфта . Эта модель сцепления является наиболее распространенной на автомобилях, находящихся в обращении. С точки зрения работы фрикционная муфта состоит из маховика, который опирается на первичный вал коробки передач с помощью бронзовой втулки. Кроме того, в зависимости от типа охлаждения фрикцион охлаждается маслом.
  • Гидравлическое сцепление. Этот тип системы сцепления позволяет двигателю передавать крутящий момент при достижении определенной частоты вращения. Таким образом, мощность передается центробежным насосом, который сообщается с турбиной.
  • Электромагнитная муфта . Также известная как муфта с электроусилителем, она работает по принципу электромагнита, то есть электромагнитное поле создается железным сердечником и катушкой для передачи крутящего момента.

 

Детали системы сцепления

Система сцепления состоит из ряда элементов, которые составляют полную передачу, в которой педаль сцепления и рычаг переключения передач с первого взгляда являются двумя частями. Что касается деталей сцепления, мы опишем наиболее распространенный тип — фрикционную муфту. Можем выделить следующее:

  • Маховик . Эта плоская круглая деталь вращается при запуске двигателя. Маховик, прикрепленный болтами к коленчатому валу (базовому компоненту, на котором построен двигатель внутреннего сгорания), вращается вместе с двигателем.
  • Нажимная пластина или ступица . Прикрученная к маховику ступица или нажимной диск отвечает за синхронное вращение. Он также состоит из диафрагмы или пружин, с помощью которых происходит выключение диска сцепления.
  • Фрикционный диск или диск сцепления . Состоит из двух сторон тормозных колодок, фрикционный диск или диск сцепления соединяется с коробкой передач с помощью вала. Когда педаль сцепления отпущена, она вращается вместе с маховиком и ступицей. При нажатии на педаль фрикционный диск выходит из зацепления, что прерывает передачу на колеса автомобиля.
  • Вилка толкателя . Последний отвечает за оказание давления на диафрагменную пружину пластины и, таким образом, освобождение или нажатие на диск. Вилка толкателя перемещается при нажатии на педаль сцепления.

Теперь, когда вы знаете, какие существуют типы сцеплений и их частей, мы рекомендуем вам посетить раздел нашего блога на веб-сайте Frenkit, где вы найдете все, что вам нужно знать для правильного обслуживания компонентов. вашего автомобиля, например, все, что касается главного цилиндра сцепления.

Независимо от того, являетесь ли вы профессионалом в автомобильной отрасли или решили самостоятельно заменить системы своего автомобиля, в Frenkit мы являемся дистрибьютором тормозных деталей для всех моделей автомобилей, как легковых, так и легковых.

Что вы думаете об этом посте? Оставьте нам свои комментарии!

Сцепления, тормоза, муфты и маховики

Что такое сцепления?

Муфта описывается как элемент машины, используемый для соединения или разъединения двух валов для передачи крутящего момента или мощности. Сцепление работает по принципу трения. Сцепление обычно используется в автомобилях для передачи мощности от ведущего вала к ведомому валу.

Функции сцепления

Ниже перечислены различные функции сцепления:

  • Запуск и прогрев двигателя.
  • Помогает включить первую передачу, чтобы начать движение автомобиля из состояния покоя.
  • Для легкого переключения передач без повреждений.
  • Для остановки автомобиля без выключения двигателя.
  • Чтобы обеспечить нагрузку на двигатель без ударов или рывков.

Типы сцепления

Ниже приведены два типа сцепления:

  • Прижимные муфты
  • Фрикционные муфты

Принудительная муфта

В принудительной муфте пара внешних муфт сцепляется, образуя жесткие соединения. Это недорогое компактное устройство, передающее вращательное движение без проскальзывания или выделения тепла. Схематическое изображение положительной муфты показано как:

CC0 1.0 | Кредиты изображений: https://themechanicalengineering.com/ | Amrit Kumar

Ниже приведены типы положительного сцепления:

  • Квадратный кулачок
  • Спиральный кулачок

Фрикционная муфта

Во фрикционной муфте включение и выключение муфты осуществляется за счет силы трения. Фрикционные муфты имеют лучшую производительность по сравнению с принудительными муфтами. Он используется практически во всех автомобилях.

Ниже приведены основные типы фрикционных муфт:

  • Однодисковая муфта
  • Многодисковая муфта
  • Конусная муфта
  • Центробежная муфта

Однодисковая муфта

В однодисковой муфте один фрикционный диск соединен с ведущим валом, а другой фрикционный диск соединен с нажимным диском и может скользить по ведомому валу. Прижимная пластина соединена с пружиной, которая прилагает осевое усилие к другой фрикционной пластине. Крутящий момент или мощность передаются от одного вала к другому, когда они вступают в физический контакт.

Обладает низкой способностью передачи крутящего момента. при работе выделяется меньше тепла; следовательно, он не требует никакой охлаждающей среды. Его еще называют сухим сцеплением.

Схематическое изображение однодискового сцепления:

Однодисковое сцепление

Многодисковое сцепление

В многодисковом сцеплении несколько фрикционных дисков работают аналогично однодисковому сцеплению. Несколько фрикционных дисков имеют большую площадь контакта, что напрямую увеличивает способность сцепления передавать крутящий момент.

Из-за большего количества фрикционных дисков многодисковая муфта быстро нагревается и требует охлаждающей жидкости. Следовательно, его также называют сцеплением мокрого типа.

Схематическое изображение многодискового сцепления показано следующим образом:

Многодисковое сцепление

Конусное сцепление

Конусное сцепление имеет два барабана; один прикреплен к ведущему валу и имеет внутреннюю поверхность трения, а другой барабан установлен на ведомом валу и имеет внешнюю поверхность трения. Когда сцепление включено, оба барабана соприкасаются и вращаются вместе.

Конусная муфта может передавать больший крутящий момент по сравнению с однодисковой муфтой. Угол конуса играет жизненно важную роль в обеспечении осевой силы, и его диапазон составляет от 12 до 15 градусов.

Схематическое изображение конусной муфты:

Конусная муфта

Центробежная муфта

Центробежная муфта, работа которой зависит от центробежной силы. Его также называют автоматическим сцеплением, потому что оно не требует педали.

В центробежном сцеплении ступица, содержащая несколько колодок, через пружину соединена с коленчатым валом двигателя, и каждая колодка имеет внешнюю фрикционную поверхность. Эти башмаки соприкасаются с барабаном на определенной скорости и передают на него мощность.

Схематическое представление центробежной муфты показано следующим образом:

Центробежная муфта

Тормоз

Тормоз представляет собой механический компонент, используемый для замедления движущегося транспортного средства, приводящего его к остановке за счет приложения силы трения. Тормоз преобразует кинетическую энергию транспортного средства в тепловую форму и рассеивает ее в природе.

Типы тормозов

Ниже приведены основные типы тормозов:

  • Механический тормоз
  • Гидравлический тормоз
  • Пневматические и вакуумные тормоза

Механический тормоз

В механическом тормозе машина замедляется за счет использования фрикционного материала против движения. Тормоз на основе трения генерирует тепло и небольшой шум. Механические тормоза широко используются в различных машинах, автомобилях и др. Механический тормоз дополнительно классифицируется как:

  • Барабанный тормоз
  • Дисковый тормоз

Барабанный тормоз

В барабанном тормозе ведущее колесо прикреплено к барабану, а фрикционные тормозные колодки используются для контакта с вращающимся барабаном для операция торможения. При нажатии на педаль тормоза кулачок поднимает тормозные колодки у барабана напротив их поворотных концов, а стопорная пружина используется для возврата тормозных колодок в исходное положение после отпускания педали тормоза.

Схематическое изображение барабанного тормоза:

Барабанный тормоз

Дисковый тормоз

В дисковом тормозе диск из высококачественного чугуна установлен вместе с колесом. Диск также соединен с дисковым суппортом, имеющим пару накладок из материала с высоким коэффициентом трения. При нажатии на педаль фрикционные накладки суппорта удерживают диск и освобождаются при отпускании педали. В настоящее время он в основном используется в технике торможения в автомобилях.

В тормозных колодках используются различные типы фрикционных материалов в зависимости от их коэффициента трения. Накладки с высоким коэффициентом обеспечивают высокую силу трения.

Схематическое изображение дискового тормоза:

Дисковый тормоз

Гидравлический тормоз

Гидравлический тормоз работает по закону Паскаля. В гидравлических тормозах тормозные жидкости используются для работы тормозного механизма. Усилие педали передается на колесный тормоз с помощью несжимаемой жидкости под давлением. Гидравлические тормоза широко используются в тяжелой технике или транспортных средствах.

Пневматические и вакуумные тормоза

В пневматических тормозах для приведения в действие тормозной системы используется сжатый воздух. В этой системе, когда педаль тормоза нажата, сжатый воздух вращает тормозной кулачок, который прижимает тормозную колодку к барабану. Он также широко используется в тяжелых транспортных средствах, таких как грузовики, поезда и другие.

В вакуумной тормозной системе торможение осуществляется за счет создания вакуума со стороны поршня. В этой системе вакуум присутствует с обеих сторон поршня, и когда тормозная лопатка нажимает, атмосферное давление прикладывается к одной стороне поршня и, толкая его с другой стороны, создает эффект торможения.

Муфты

Муфта представляет собой механическое устройство, используемое для соединения двух валов. Основная цель соединения состоит в том, чтобы объединить две вращающиеся части машин, допуская при этом некоторое конечное движение.

Схематическое изображение муфт показано следующим образом:

Муфты

Классификация муфт

Ниже приводится классификация муфт:

  • Жесткие муфты
  • Гибкие муфты

Жесткие2 муфты хорошо сочетаются с жесткими муфтами 90 — выровненные валы. Это просто и экономично. Ниже приведены различные типы жесткой муфты:

  • Втулочная или муфтовая муфта
  • Хомутная или разъемная муфта
  • Фланцевая муфта

Гибкие муфты

Эластичные муфты используются для передачи крутящего момента с одного вала на другой, когда два вала имеют поперечное и угловое смещение. Ниже приведены различные типы эластичных муфт:

  • Втулочная муфта
  • Универсальная муфта
  • Муфта Oldham

Маховики

Маховик описывается как механическое вращающееся устройство, которое собирает или расширяет энергию. Обычно он сделан из стали и вращается на соответствующих подшипниках.

Он используется в машинах или автомобилях в качестве резервуара, который накапливает энергию во время избыточной подачи энергии и высвобождает накопленную энергию, когда это необходимо.

Схематическое изображение маховика показано следующим образом:

Маховик

Функции маховика

Ниже приведены основные функции маховика:

  • Он поглощает энергию, когда его скорость увеличивается, и высвобождает энергию, когда его скорость уменьшается.
  • Уменьшает колебания скорости.
  • Уравновешивает колебания скорости двигателя во время каждого рабочего цикла.

Применение маховиков

Ниже приведены различные области применения маховиков:

  • Маховик помогает поддерживать постоянную угловую скорость коленчатого вала двигателя.
  • Используется для обеспечения непрерывной подачи энергии в системах с непостоянным источником энергии.
  • Используется в клепальном станке для хранения энергии двигателя.
  • Также используется для ориентации спутниковых приборов.

Распространенные ошибки

Ниже приведены распространенные ошибки, совершаемые учащимися:

  • Иногда учащиеся забывают, что сцепление и тормоза работают по принципу трения.
  • Иногда студенты забывают, что гидравлический тормоз работает по закону Паскаля.
  • Иногда студенты путают принудительную муфту и муфты.
  • Иногда учащиеся не понимают функции маховика.

Контекст и приложения

Тема сцеплений, тормозов, муфт и маховиков имеет большое значение на различных курсах и профессиональных экзаменах на уровне бакалавриата, магистратуры, аспирантуры и докторантуры. Например:

  • Бакалавр технологии в области машиностроения
  • Бакалавр технологии в области автомобилестроения
  • Магистр технологии в области машиностроения
  • Доктор философии в области автомобилестроения
  • Крутящий момент или передача мощности
  • Автомобильные компоненты
  • Конструкция вала
  • Основные автомобильные термины

Практические задачи

1 кв. Какое из следующих сцеплений не требует охлаждающей жидкости?

  1. Однорезовая сцепление
  2. Многоплановая муфта
  3. Конопочечное сцепление
  4. Ни один из этих

Правильный вариант : (a)

ЭКСПЛУАТАЦИЯ: во время трансмиссии питания, более тепло. Многодисковая муфта и коническая муфта требуют системы смазки для охлаждения. Благодаря охлаждению или смазке коэффициент трения будет снижен. Для одной пластины не требуется система охлаждения.

Q2. Обычно диапазон угла конуса муфты конуса составляет ______?

  1. 15–20 градусов
  2. 4–15 градусов
  3. 5–10 градусов
  4. с 12 до 15 градусов

Правильный вариант: (D)

Объяснение: дается как 12 градусов. Угол конуса может варьироваться от 12 до 15 градусов. Она всегда должна быть выше 8 градусов.

Q3 . По какому закону работает гидравлический тормоз?

  1. Электромагнитный закон
  2. Закон Паскаля
  3. Закон Фарадея
  4. Сохранение импульса

Правильный вариант: (b)

Объяснение: Generators Работа над ElectromAgnetitic. Электрический трансформатор работает по закону Фарадея. Ракетные и реактивные двигатели работают по закону сохранения импульса. Но гидравлический тормоз работает по закону Паскаля.

4 квартал . Что из перечисленного не является упругой муфтой?

  1. Универсальные муфты
  2. Олдхэмские соединения
  3. Фланковые муфты
  4. Связывание с кустарником

Правильный вариант: (c)

. муфты. Но фланцевые муфты, как правило, постоянно закрепляются на конце каждого вала. Это не гибкие муфты.

Q5 . Что из перечисленного является функцией маховика?

  1. Уменьшает колебания
  2. Увеличение скорости
  3. Увеличивает кинетическую энергию
  4. снижение кинетической энергии

Правильный вариант: (A)

Объяснение: .

Сцеплен

Добавить комментарий

Ваш адрес email не будет опубликован. Обязательные поля помечены *